NCLEX questions

¡Supera tus tareas y exámenes ahora con Quizwiz!

The nurse is planning a teaching session with a client about a new diagnosis of congestive heart failure. The nurse and client do not speak the same language. The nurse arranges for an interpreter to be present for the teaching session. Which nursing concept is this an example of? a.) quality improvement b.) client advocacy c.) Confidentiality d.) informed consent

b.) client advocacy

The nurse is teaching a client about prevention of recurrent kidney stones. Which statement by the client would indicate that additional teaching is needed? a.)"I will monitor the color of my urine." b.)I will follow a low-calcium diet and avoid dairy products." c.)"I will contact my health care provider if I am having difficulty urinating." d.)"I will make sure I drink plenty of water throughout the day."

b.)I will follow a low-calcium diet and avoid dairy products."

The nurse is concerned that a client with a history of anorexia nervosa may be experiencing a recurrence of the condition. Which of the following findings would support the nurse's concern? Select all that apply. a.)Elevated blood pressure b.)Weight loss c.)Lack of exercise d.)Recent hair loss e.)Constipation.

b.)Weight loss d.)Recent hair loss e.)Constipation.

The nurse is assessing a postoperative client following femoral popliteal bypass surgery for peripheral arterial disease. The surgical dressing is saturated with bright red blood. Which action should the nurse take first? a.)Increase the rate of the normal saline IV infusion b.)Apply a pressure dressing over the surgical dressing c.)Stop the heparin sodium infusion d.)Obtain a set of vital signs

c.)Stop the heparin sodium infusion

The nurse is evaluating a client with status asthmaticus. Which finding best indicates that interventions were effective? a.)The client's respiratory rate is 20. b.)The client denies shortness of breath. c.)The client's pulse oximeter reads 94%. d.)The client's wheezes have decreased in intensity.

c.)The client's pulse oximeter reads 94%.

Chadwick's signs

a bluish-purple coloration of the cervix and vaginal walls. It develops at 6 to 8 weeks of gestation and is caused by an increased blood supply to the area.

a client is being discharged with a prescription for warfarin. The client asks "May I take aspirin with this medication? It helps my arthritis." Which response by the nurse is appropriate to address the client's concern? a.) "Avoid aspirin because it can increase the bleeding effects of warfarin." b.)"When you take the aspirin, do not take the warfarin that day." c.)"Take the warfarin in the morning and the aspirin at night." d.)"Use about half the recommended dose of aspirin."

a.) "Avoid aspirin because it can increase the bleeding effects of warfarin."

The nurse is providing information to a pregnant client about the potential risks of an amniocentesis. Which risk factors shall the nurse include? Select all that apply. a.) Preterm labor b.) Premature rupture of membranes c.) Ectopic pregnancy d.) Preeclampsia e.) Spontaneous abortion f.) Increase in blood glucose levels

a.) Preterm labor b.) Premature rupture of membranes e.) Spontaneous abortion

The nurse in an urgent care clinic is discharging a client diagnosed with pyelonephritis due to a urinary tract infection. Which instruction is most important to include? a.)"Notify your health care provider if you do not start feeling better in a few days." b.)"Schedule an appointment for a repeat urinalysis in one week." c.)"Monitor your blood pressure at home daily." d.)"Increase your fluid intake to 3 to 4 liters a day."

a.)"Notify your health care provider if you do not start feeling better in a few days."

The nurse is caring for a client recently diagnosed with hypothyroidism. Which client statement indicates that additional teaching is needed? a.)"Once I feel better, I won't need to take this medication every day." b.)"Within the next few weeks, I should likely feel less fatigued." c.)"I will have to have blood tests done regularly." d.)"I might experience anxiety or problems with sleeping."

a.)"Once I feel better, I won't need to take this medication every day."

The preoperative nurse is reviewing the medical record of a client who is scheduled for a surgery with general anesthesia. Which information in the client's medical record should be reported to the health care provider prior to surgery? Select all that apply. a.)Anticoagulant use b.)Vitiligo c.)Latex allergy d.)Anxiety e.)Family history of malignant hyperthermia

a.)Anticoagulant use c.)Latex allergy e.)Family history of malignant hyperthermia

The nurse is caring for a client who presents with polyuria, polydipsia and a urine specific gravity of 1.002. The nurse suspects that the client is experiencing diabetes insipidus. Which risk factors would support this diagnosis? Select all that apply. a.)History of recent surgery b.)Current use of lithium c.)Recent neurologic injury d.)History of radiation treatment f.)History of pulmonary disease

a.)History of recent surgery b.)Current use of lithium c.)Recent neurologic injury d.)History of radiation treatment

The nurse is caring for a client who is taking leuprolide for endometriosis. The nurse should monitor the client for which side effects? Select all that apply. a.)Hot flashes b.)Anorexia c.)Vaginal dryness d.)Amenorrhea e.)Increased fertility f.)Emotional lability

a.)Hot flashes c.)Vaginal dryness d.)Amenorrhea f.)Emotional lability

he home health nurse is reviewing the plan of care for a client experiencing acute attacks of Ménière's disease. What is the priority intervention for this client? a.)Instruct the client not to drive a motor vehicle. b.)Encourage bland foods and noncarbonated fluids. c.)Provide assistance with bathing and dressing. d.)Communicate clearly and use visual aids.

a.)Instruct the client not to drive a motor vehicle.

The nurse is assessing the newborn of a mother who tested positive for heroin. Which assessment finding should the nurse anticipate for this infant? a.)Irritability b.)Lethargy with excessive sleepiness c.)Central nervous system depression d.)Large for gestational age

a.)Irritability

The nurse is caring for a client who has suspected Cushing's disease. The nurse should monitor for which potential symptoms? Select all that apply. a.)Large fat pads on the back and shoulders b.)Polyuria and polydipsia c.)History of pathologic fractures d.)Tachycardia and panic attacks e.)Changes in visual acuity

a.)Large fat pads on the back and shoulders c.)History of pathologic fractures

The nurse is caring for a client with a pressure ulcer on their heel that is covered with hard, dry black tissue. Which action should the nurse take? a.)Leave the pressure ulcer as is. b.)Apply a wet-to-dry dressing. c.)Remove the eschar. d.)Apply a hydrocolloid dressing.

a.)Leave the pressure ulcer as is.

A client with advanced liver disease has been taking rifaximin. Which assessment finding would indicate that the medication is being effective? a.)Less confusion b.)Increased appetite c.)Less jaundice d.)Less edema

a.)Less confusion

A client has been diagnosed with hypothyroidism. Which medication should the nurse administer to treat the client's bradycardia? a.)Levothyroxine b.)Epinephrine c.)Adenosine d.)Atropine

a.)Levothyroxine

The office nurse is discussing how to prevent an acute gouty attack with a client who has gout. Which actions should the nurse recommend to the client? Select all that apply. a.)Limit the consumption of alcohol. b.)Make sure to drink at least 2,000 mL of water daily. c.)Limit the intake of shellfish and red meats. d.)Implement stress reduction techniques. e.)Take the prescribed prednisone regularly.

a.)Limit the consumption of alcohol. b.)Make sure to drink at least 2,000 mL of water daily. c.)Limit the intake of shellfish and red meats. d.)Implement stress reduction techniques.

A community health nurse has been caring for a woman who is 22 weeks pregnant and has a history of morbid obesity, asthma and hypertension. Which of these lab reports should be communicated to the primary health care provider immediately? a.)Magnesium 0.8 mEq/L and Creatinine 3 mg/dL b.)Hematocrit 35% and platelets 200,000/mm3 c.)Hemoglobin 13 g/dL and Calcium 5.1 mg/dL d.)Blood urea nitrogen 28 mg/dL and Glucose 225 mg/dL

a.)Magnesium 0.8 mEq/L and Creatinine 3 mg/dL

The nurse is developing a plan of care for a client with benign prostatic hyperplasia. Which nursing interventions should the nurse include for this client? Select all that apply. a.)Monitor for bladder distention. b.)Limit caffeinated and alcoholic beverages. c.)Void every 1 to 2 hours to empty the bladder. d.)Catheterize as needed for post-void residual urine. e.)Calculate accurate intake and output.

a.)Monitor for bladder distention. b.)Limit caffeinated and alcoholic beverages. d.)Catheterize as needed for post-void residual urine.

The nurse is caring for a client who has been diagnosed with syndrome of inappropriate antidiuretic hormone (SIADH). Which interventions are appropriate for this client? Select all that apply. a.)Monitoring of intake and output b.)Administration of vasopressin c.)Implementation of a fluid restriction d.)Implementation of a low-sodium diet e.)Administration of a loop diuretic

a.)Monitoring of intake and output c.)Implementation of a fluid restriction e.)Administration of a loop diuretic

The nurse is caring for a female client with a body mass index of 45. Which conditions should the nurse plan to discuss with the client due to the risks associated with her weight? Select all that apply. a.)Obstructive sleep apnea b.)Coronary artery disease c.)Gallstones d.)Breast cancer e.)Chronic obstructive pulmonary disease f.)Hyperthyroidism

a.)Obstructive sleep apnea b.)Coronary artery disease c.)Gallstones d.)Breast cancer

The home health nurse is educating the parent of a child who has a chronic condition that limits mobility. Which statement best describes the effects of immobility in children? a.)Immobilized children quickly develop confusion and mental status changes b.)Immobility promotes independence and self-reliance in children c.)The physical effects of immobility are similar in both children and adults d.)Children are more susceptible than adults to the multisystem effects of immobility

c.)The physical effects of immobility are similar in both children and adults

The nurse is reviewing the assessment data of a client suspected of having diabetes insipidus. Which of the following findings should the nurse expect after a water deprivation test? a.)Decreased serum potassium b.)Rapid protein excretion c.)Unchanged urine specific gravity d.)Increased edema and weight gain

c.)Unchanged urine specific gravity

The nurse is assessing a client who just returned to the medical surgical unit after a segmental lung resection surgery. During the assessment, the client is coughing and clearing their throat. What is the first action the nurse should take? a.)Apply the pulse oximeter and monitor oxygen saturation. b.)Administer the PRN pain medication. c.)Assist the client to turn, deep breathe and cough. d)Suction excessive tracheobronchial secretions.

d)Suction excessive tracheobronchial secretions.

a female client is admitted for a breast biopsy. She says, tearfully, to the nurse, "If this turns out to be cancer and I have to have my breast removed, my partner will never come near me." Which of these statements would be the best response by the nurse? a.) "I hear you saying that you have a fear of the loss of love." b.)"You sound concerned that your partner will reject you." c.) "Are you wondering about the effects on your sexuality?" d.) "Are you worried that the surgery will lead to changes?"

d.) "Are you worried that the surgery will lead to changes?"

The nurse is teaching a client with chronic renal failure about their medications. The client questions the purpose of taking aluminum hydroxide. How should the nurse respond? a.) "It increases your urine output." b.) "It is taken to control gastric acid secretions." c.) "It will reduce your blood's calcium levels." d.) "It decreases your blood's phosphate levels."

d.) "It decreases your blood's phosphate levels."

The parents of a 3-year-old toddler ask the nurse how long their child will have to sit in a child car seat while in an automobile. Which is the best response by the nurse? a.) "The child must be 5-years-old to use a regular seat belt." b.) "The child can use a regular seat belt when they can sit still." c.) "The child must reach a height of 50 inches (127 cm) to use a regular seat belt." d.) "The child should use a car seat for as long as possible."

d.) "The child should use a car seat for as long as possible."

The nurse is providing safety and accident prevention instructions to the parents of 4-year-old twins. Which statement by the parents indicates a correct understanding of the instructions? a.) "Accidents can happen. We will make sure to call 911 right away." b.) "It is normal for little children to be curious and want to explore." c.) "The children are old enough to sit in the car without a child safety seat." d.) "We are having a fence installed around the backyard pool."

d.) "We are having a fence installed around the backyard pool."

A client is admitted with severe injuries resulting from an auto accident. The client's vital signs are BP 120/50 mmHg, pulse rate 110 bpm, and respiratory rate of 28 breaths per minute. Which action should the nurse complete first? a.) Initiate the ordered intravenous therapy. b.) Institute continuous cardiac monitoring. c.) Initiate continuous blood pressure monitoring. d.) Administer oxygen as ordered.

d.) Administer oxygen as ordered.

A newborn baby that was delivered at home without a birth attendant is admitted to the hospital for observation. The baby's initial temperature is 95° F (35° C). The nurse should recognize that the newborn is at risk for which complication? a.)Hyperglycemia b.)Metabolic alkalosis c.)Lowered basal metabolic rate d.) Hypoxemia

d.) Hypoxemia

The nurse performs a heel stick for a blood glucose check on a 1-hour-old, full-term newborn who weighed 9 pounds (4.1 kg) at birth. The serum glucose reading is 45 mg/dL. Which action should the nurse take? a.) Check the pulse oximetry reading. b.) Start an IV infusion of 5% dextrose. c.) Ask another nurse to perform a second blood glucose check. d.) Repeat the test in two hours.

d.) Repeat the test in two hours.

A parent asks about expected motor skill development for their 3-year-old child. Which activity is considered a typical motor skill for that age? a.) Tying shoelaces b.) Jumping rope c.) Playing hopscotch d.) Riding a tricycle

d.) Riding a tricycle

The nurse is planning care for a 2-year-old hospitalized child. Which issue will produce the most stress at this age? a.)Loss of control b.)Fear of pain c.)Bodily injury d.) Separation anxiety

d.) Separation anxiety

When planning care for a client at risk for pulmonary embolism, the nurse shall make which intervention a priority? a.)Instruct client on how to use the incentive spirometer. b.)Encourage client to cough and deep breathe. c.)Maintain client on bedrest. d.)Apply sequential compression devices to the legs.

d.)Apply sequential compression devices to the legs.

A pregnant client at 34 weeks gestation is diagnosed with a pulmonary embolism. Which of these medications should the nurse plan to administer? a.) Oral warfarin b.)Oral low-dose aspirin c.)Subcutaneous enoxaparin d.)Intravenous heparin

d.)Intravenous heparin

The nurse provides regular mouth care to the hospice client who is actively dying at home. The family wants to know why the doctor doesn't order intravenous (IV) fluids since the client's mouth seems so dry. What information can the nurse provide to accurately answer this question? a.)The client will need to have an indwelling catheter inserted if an IV is started b.)Intravenous hydration will increase episodes of delirium c.)The client will need to be hospitalized if an IV is started d.)Intravenous hydration can delay death

d.)Intravenous hydration can delay death

The nurse is caring for a client with osteoporosis who has been prescribed alendronate. When providing care, which intervention would be a priority? a.)Administer the alendronate 30 to 60 minutes before the client eats. b.)Encourage the client to increase their intake of vitamin D. c.)Monitor the client's serum calcium levels. d.)Notify the health care provider if the client reports jaw pain.

d.)Notify the health care provider if the client reports jaw pain.

The recovery room nurse is caring for an infant following the surgical correction of a ventricular septal defect. Which nursing assessment is the priority? a.)Auscultate for pulmonary congestion b.)Blanch nail beds for color and refill c.)Monitor for the equality of peripheral pulses d.)Observe for postoperative dysrhythmias

d.)Observe for postoperative dysrhythmias

The nurse is evaluating the effectiveness of therapy for a client who received albuterol via nebulizer during an acute episode of shortness of breath due to asthma. Which finding is the best indicator that the therapy was effective? a.)Respiratory rate is 16 breaths/minute. b.)Accessory muscle use has decreased. c.)No wheezes are audible. d.)Oxygen saturation is greater than 90%.

d.)Oxygen saturation is greater than 90%.

The nurse in a behavioral health inpatient unit is observing a female client who has been diagnosed with obsessive-compulsive disorder. Which behavior supports this diagnosis? a.)The client prefers to interact with female staff members. b.)The client verbalizes suspicions about thefts on the unit. c.)The client exhibits repetitive, involuntary movements. d.)The client is seen washing her hands every 15 minutes.

d.)The client is seen washing her hands every 15 minutes.

The nurse is caring for a client with chronic atrial fibrillation. Which drug does the nurse expect to administer to prevent a common complication of this condition? a.)Carvedilol b.)Diltiazem c.)Lidocaine d.)Warfarin

d.)Warfarin

indication of accurate I&O

Daily weights

A client is taking diphenhydramine for seasonal allergic rhinitis. The nurse should reinforce teaching for the client about which possible side effects? Select all that apply. a.)Urinary retention b.)Drowsiness c.)Urinary frequency d.)Dry mouth e.)Constipation

a.)Urinary retention b.)Drowsiness d.)Dry mouth e.)Constipation

The nurse is assessing a client who was admitted with suspected Guillain-Barré syndrome. Which assessment findings should the nurse expect? Select all that apply. a.)Seizures b.)Hyporeflexia c.)Hypotonia d.)Paresthesia e.)Diarrhea f.)weakness

b.)Hyporeflexia c.)Hypotonia d.)Paresthesia f.)weakness

The nurse receives a client who was transported to the emergency department for severe hypertension. Which finding requires immediate action by the nurse? a.)Crackles in the lung bases b.)Weakness in the left arm c.)Cough with frothy, pink sputum d.)Jugular vein distension

b.)Weakness in the left arm

The nurse is reviewing the medical record of a client with acute pancreatitis. The nurse should recognize which information as the most likely risk factor for the client's illness? a.)Gallstones b.)Obesity c.)Diabetes d.)Intravenous heroin use

a.)Gallstones

the nurse is assessing a 72-year-old client with a full-leg cast on his left leg three days after cast application and finds bilateral pedal edema. Based on this finding, what condition should the nurse consider? a.)Heart failure b.)Thrombophlebitis c.)Infection d.)Compartment syndrome

a.)Heart failure

A client diagnosed with renal calculi is admitted to the medical surgical unit. Which intervention should the nurse implement first? a.)Assess the client's pain. b.)Monitor the client's urinary output. c.)Increase the client's oral fluid intake. d.)Review appropriate diet choices with client.

a.)Assess the client's pain.

The nurse is caring for a client in the late stages of amyotrophic lateral sclerosis. Which finding is consistent with this diagnosis? a.)Confusion b.)Shallow respirations c.)Loss of half of visual field d.)Tonic-clonic seizures

b.)Shallow respirations

The nurse is reviewing a client's medication list and notes the client takes bupropion SR 150 mg oral twice a day. Which question is appropriate for the nurse to ask concerning the purpose of this medication? a.)"Did your cravings for nicotine decrease after starting this medication?" b.)"How much weight have you gained on this medication?" c.)"Have you had any abnormal dreams while taking this medication?" d.)"After taking this medication, did your hallucinations lessen?"

a.)"Did your cravings for nicotine decrease after starting this medication?"

A client who has cirrhosis of the liver underwent a paracentesis yesterday. Today, the unlicensed assistive personnel (UAP) reports the client is lethargic and has musty-smelling breath. Which assessment should the nurse perform next? a.)Assess for flap-like tremors of the hands b.)Measure the abdominal girth changes c.)Monitor the client's clotting status d.)Auscultate the upper abdomen for bruits

a.)Assess for flap-like tremors of the hands

The charge nurse in the emergency department receives a radio call from Emergency Medical Services (EMS) stating that there has been a large structure fire with multiple victims. Which action should the charge nurse take first, before the victims start to arrive? a.) Notify the nursing supervisor and request additional staff. b.) Activate the disaster plan c.) Prepare the trauma room and lay out supplies. d.) Call for a medical evacuation helicopter to be on standby.

a.) Notify the nursing supervisor and request additional staff.

A new task force has been created at a hospital to address a recent increase in client falls. The first meeting is scheduled with members from several departments. Which statements by the nurse leader will increase meeting effectiveness? Select all that apply. a.) "Please introduce yourselves and your departments." b.) "Let's discuss when we should meet next and what information we will bring." c.) "Let's focus on the number of client falls first and then we can talk about staffing." d.)"During our meeting today, we will share the information we have on client falls." e.) "Today I will review the problem with client falls on our units." f.) "This meeting can go as long as needed to get things done."

a.) "Please introduce yourselves and your departments." b.) "Let's discuss when we should meet next and what information we will bring." c.) "Let's focus on the number of client falls first and then we can talk about staffing." d.)"During our meeting today, we will share the information we have on client falls."

A client is evaluated in the emergency department for an ankle sprain. Which discharge instructions should the nurse provide? Select all that apply. a.) "To help decrease swelling, it is important to keep your ankle elevated when you are resting." b.) "Wrap your foot and ankle tightly with the elastic wrap and leave on overnight." c.) "Rest your foot and ankle tonight but start strengthening exercises tomorrow morning." d.) "You should apply an ice pack to your ankle for 20 to 30 minutes at a time, 3 to 4 times a day." e.) "It is important to avoid standing or walking without crutches until after your follow-up visit."

a.) "To help decrease swelling, it is important to keep your ankle elevated when you are resting." d.) "You should apply an ice pack to your ankle for 20 to 30 minutes at a time, 3 to 4 times a day." e.) "It is important to avoid standing or walking without crutches until after your follow-up visit."

The nurse is conducting a community-wide seminar on childhood safety issues. Which child is at the highest risk for poisoning? a.) A 20-month-old who has just learned to climb stairs b.) A 15-year-old who likes to repair bicycles c.) A 9-month-old who stays with a babysitter five days a week d.) A 10-year-old who occasionally stays at home unattended

a.) A 20-month-old who has just learned to climb stairs

he nurse is teaching a client with cardiac disease who is taking furosemide and digoxin about foods rich in potassium. Which food choice best indicates the client understands the teaching? a.) A baked potato b.) An apricot c.)A small orange d.) A small banana

a.) A baked potato

The nurse is caring for a group of clients on a medical-surgical unit. The nurse understands that which situations would require hand hygiene? Select all that apply. a.) After changing a client's sterile dressing b.) After making an entry in a client's medical record c.) Before and after taking a break and eating lunch d.) Prior to entering a client's room e.)After wiping down a client's bedside table

a.) After changing a client's sterile dressing eating lunch d.) Prior to entering a client's room e.)After wiping down a client's bedside table

Which of these activities are examples of primary prevention activities? Select all that apply. a.) An exercise class b.) Rehabilitation c.)Breast self-exam d.) Car seat installation education e.) Cholesterol screening f.) Vaccination

a.) An exercise class d.) Car seat installation education f.) Vaccination

The nurse is providing burn prevention and home safety education to parents of small children. Which safety measures should the nurse include in the teaching plan? Select all that apply. a.) Check seatbelt buckles before placing a child in a car seat. b.)Replace smoke detector batteries once per year. c.)Set the water heater temperature to 130° F (54.4° C). d.) Create a fire escape plan and practice it with the family. e.)Turn pot handles towards the center of the stove when cooking.

a.) Check seatbelt buckles before placing a child in a car seat. d.) Create a fire escape plan and practice it with the family. e.)Turn pot handles towards the center of the stove when cooking.

The nurse is assessing a client during the first stage of labor. Which action is correct when evaluating the characteristics of uterine contractions? a.) Evaluate intensity by pressing fingertips into the uterine fundus. b.) Assess uterine contractions every 30 minutes throughout the first stage of labor. c.) Place a hand on the abdomen below the umbilicus and palpate uterine tone with fingertips. d.) Determine frequency by timing the end of one contraction until the end of the next contraction.

a.) Evaluate intensity by pressing fingertips into the uterine fundus.

The nurse is completing a head-to-toe assessment on a client. The nurse notes a pulsating mass in the client's periumbilical area. Which assessment is appropriate for the nurse to perform? a.)Auscultate b.)Percuss c.)Measure the length d.)Palpate

a.)Auscultate

The nurse hears a scream coming from a client's room. When entering the room, the nurse finds the client lying on the floor beside the bed. Which of the following actions should the nurse take? Select all that apply. a.) Observe the client for abnormal leg rotation. b.)Determine the client's level of consciousness. c.)Take the client's vital signs. d.)Notify the client's provider about the incident. e.)Report the incident to the facility's lawyer. f.)Place the client in physical restraints to prevent another fall.

a.) Observe the client for abnormal leg rotation. b.)Determine the client's level of consciousness. c.)Take the client's vital signs. d.)Notify the client's provider about the incident.

a client comes to the community health clinic with symptoms of gonorrhea. Which intervention should the nurse implement first? a.) Obtain information about the client's recent sexual encounters. b.)Collect a urethral swab from the client. c.)Instruct the client to notify past sexual partners. d.)Discuss the risk of infertility with the client.

a.) Obtain information about the client's recent sexual encounters.

The nurse in an intensive care unit is reviewing the laboratory results for several clients. Which laboratory result indicates that the client has a partially compensated metabolic acidosis? a.) PaCO2 of 30 mmHg b.)Chloride of 100 mEq/L c.)HCO3 of 28 mEq/L d.)pH of 7.48

a.) PaCO2 of 30 mmHg

Which action shall the nurse take to preserve the client's right to autonomy? a.) Providing the client with requested information to make an informed decision. b.) Answering all the client's questions about their prognosis truthfully. c.) Assisting the client to the bathroom in a timely manner. d.) Having the client set up a password for their electronic health record.

a.) Providing the client with requested information to make an informed decision.

The nurse is caring for a client with schizophrenia, who has an order for haloperidol 5 mg PO every four hours as needed. Which behaviors justify the use of this chemical restraint? Select all that apply. a.) The client is expressing paranoid delusions. b.) The client is refusing to participate in unit group activities. c.) The client is crying after a difficult family meeting. d.) The client is verbalizing a plan to harm another client. e.) The client is experiencing command hallucinations.

a.) The client is expressing paranoid delusions. d.) The client is verbalizing a plan to harm another client. e.) The client is experiencing command hallucinations.

The nurse is collecting baseline data on a 14-month-old child during a wellness visit in the primary care provider's office. Which of the following measurement methods are correct? Select all that apply. a.) The nurse places the tape measure around the child's head at the widest part of the frontal and occipital bones. b.) The nurse places the child on an infant platform scale in either a sitting or supine position. c.) The nurse measures the child's chest circumference by placing the measurement tape around their chest at the nipple line. d.) The nurse counts the child's pulse by placing one finger on the radial artery for a full minute. e.) The nurse measures the child's height while the child stands against the wall supported by the parent.

a.) The nurse places the tape measure around the child's head at the widest part of the frontal and occipital bones. b.) The nurse places the child on an infant platform scale in either a sitting or supine position. c.) The nurse measures the child's chest circumference by placing the measurement tape around their chest at the nipple line.

The nurse is caring for a client who is confused and has repeatedly attempted to pull out their intravenous lines and feeding tube. The nurse receives an order from the health care provider (HCP) to apply soft wrist restraints. Which actions by the nurse are appropriate? Select all that apply. a.) Tie the wrist restraints using quick-release knots. b.) Conduct a thorough physical assessment of the client. c.) Explain the rationale for the use of restraints to the client. d.) Document that alternative interventions were attempted. e.) Release the restraints and provide care every four hours. f.) Call the HCP every 48 hours for a new restraint order.

a.) Tie the wrist restraints using quick-release knots. b.) Conduct a thorough physical assessment of the client. c.) Explain the rationale for the use of restraints to the client. d.) Document that alternative interventions were attempted.

The nurse on an inpatient medical unit is caring for a client who is in the advanced stage of multiple myeloma. Which intervention should the nurse include in the plan of care? a.) Use careful repositioning techniques. b.) Monitor the client for hypokalemia. c.) Place the client in protective isolation. d.) Administer dicturetics as ordered.

a.) Use careful repositioning techniques.

A client has been admitted to an inpatient behavioral health unit for severe depression and suicidal threats. The client has been placed on suicide precautions. The nurse should be aware that the danger of the client attempting suicide is greatest during what period of time? a.) When the client's mood or energy level improves. b.) During the night shift when staffing is limited. c.) After a visit from the client's estranged partner. d.) At the time of the client's greatest despair.

a.) When the client's mood or energy level improves.

the parent of a 4-month-old infant asks the nurse about how to protect the child from sunburn. Which of these statements is the best advice about sun protection for infants? a.)"Dress the infant in lightweight long pants, long-sleeved shirts and brimmed hats." b.)"You should keep the baby inside unless it's cloudy outside." c.)"Sunscreen should not be used on children." d.)"Liberally apply a sunscreen with the minimum sun protective factor of 15 all over the child's body."

a.)"Dress the infant in lightweight long pants, long-sleeved shirts and brimmed hats."

The nurse is admitting a male client who is newly diagnosed with a frontal lobe brain tumor. Which statement by the client's spouse would support this diagnosis? a.)"I find the mood swings hard to deal with." b.)"His breathing rate is usually below 12." c.)"He has a hard time reading small print." d.) "It seems that he has to urinate more frequently."

a.)"I find the mood swings hard to deal with."

The home health nurse is reviewing the medical record of a client with closed-angle glaucoma in both eyes. Which statement by the client would support this diagnosis? a.)"I have to turn my head to see around the room." b.)"I have constant blurred vision." c.)"I can't see out of my left eye." d.)"I have specks floating in my eyes."

a.)"I have to turn my head to see around the room."

The nurse is providing discharge teaching for a client after cataract surgery of the left eye. Which statements by the client indicate an understanding of the teaching? Select all that apply. a.)"I will call the surgeon if the pain is intense." b.)"I will drive very carefully today." c.)"I will follow the instructions for the eye drops." d.)"I will not rub, press on or scratch my eye." e.)"I will call the surgeon if my eye is bloodshot."

a.)"I will call the surgeon if the pain is intense." c.)"I will follow the instructions for the eye drops." d.)"I will not rub, press on or scratch my eye."

A nurse is caring for a client recently diagnosed with Addison's disease. While the nurse is reinforcing education, which statement by the client indicates the need for additional teaching? a.)"I will take my methylprednisolone when I start to feel sick." b.)"I will wash my hands often to prevent infections." c.)"I will eat six small meals a day instead of three." d.)"I may experience changes in my mood."

a.)"I will take my methylprednisolone when I start to feel sick."

The nurse is discharging a client from the hospital who was admitted for acute exacerbation of chronic obstructive pulmonary disease. Which statement by the client indicates that teaching was effective?a.)"I will use my spacer each time I use my inhaler." b.)"I will switch from regular to electronic cigarettes." c.)I will make sure to get the pneumonia vaccine every year." d.)"I will eat foods low in calories and protein."

a.)"I will use my spacer each time I use my inhaler."

The parent of a 5-year-old child is concerned about an outbreak of measles in the community. The nurse understands that additional education about immunizations is needed when the parent makes which of the following statements? Select all that apply. a.)"My child should have passive immunity from the vaccine I had as a child." b.)"My child should receive a second dose of the measles vaccine now." c.)"My child is unlikely to get measles because of their first vaccine at age one." d.)"If a child develops a rash, the risk of spreading measles is gone." e.)"We should avoid playing with children with high fevers."

a.)"My child should have passive immunity from the vaccine I had as a child." d.)"If a child develops a rash, the risk of spreading measles is gone."

The nurse at the outpatient surgery center is speaking with a client who is scheduled for a colonoscopy the next morning. Which information about the procedure should the nurse make sure to include? Select all that apply. a.)"Remember to stop eating any food six hours before you come to the center." b.)"You should only consume clear liquids for the next 12 to 24 hours." c.)"Make sure to drink the entire bowel preparation liquid." d.)"You will be required to lay still for 6 to 8 hours after the procedure." e)"You will have an intravenous catheter inserted prior to the procedure."

a.)"Remember to stop eating any food six hours before you come to the center." b.)"You should only consume clear liquids for the next 12 to 24 hours." c.)"Make sure to drink the entire bowel preparation liquid." e)"You will have an intravenous catheter inserted prior to the procedure."

The nurse is speaking with the parents of a 5-year-old boy who is diagnosed with hemophilia A. The parents recently underwent genetic counseling that showed that the mother is a carrier and the father is unaffected. The parents are asking the nurse what the chances are of having another child with this genetic disorder. How should the nurse respond? a.)"There is a 25% probability that daughters will be a carrier of this disease." "b.)All daughters will be carriers of this disease." c.)"All of your male children will have this disease." d.)"There is a 50% probability that another male child would have this disease."

a.)"There is a 25% probability that daughters will be a carrier of this disease."

A client with benign prostatic hypertrophy has been prescribed tamsulosin. Which statement by the nurse correctly describes how this medication works? a.)"This medication will improve the flow of urine." b.)"This drug will eliminate your nocturia." c.)"Your libido will increase with this medication." d.)"This medication will shrink your enlarged prostate gland."

a.)"This medication will improve the flow of urine."

The nurse is educating a group of individuals about how to prevent hepatitis B and C. Which statement by the nurse would best describe prevention of these two diseases? a.)"You should use protection when engaging in sexual intercourse." b.)"You can receive a yearly vaccination to prevent the diseases." c.)"You should talk to your health care provider when traveling internationally." d.)"You can eat fresh fruit picked from the tree without the need to wash."

a.)"You should use protection when engaging in sexual intercourse."

he nurse should monitor which clients who may be at-risk for the development of acute kidney injury? Select all that apply. a.)A client admitted with an acute myocardial infarction b.)A client who received multiple blood transfusions c.)A client with a history of syndrome of inappropriate diuretic hormone d.)A client with a history of cirrhosis e.)A client recovering from septic shock

a.)A client admitted with an acute myocardial infarction b.)A client who received multiple blood transfusions e.)A client recovering from septic shock

The nurse is reviewing the medical record of a client who has been diagnosed with systemic lupus erythematosus. The nurse would expect which findings associated with this disease? Select all that apply. a.)A temperature of 100.6° F (38° C) b.)A red, raised rash on the face c.)Generalized weakness d.)Polydipsia for the last month e.)A recent ten pound weight gain f.)Reports of pain in the hands and knees

a.)A temperature of 100.6° F (38° C) b.)A red, raised rash on the face c.)Generalized weakness f.)Reports of pain in the hands and knees

The nurse is evaluating an adult client who is receiving continuous enteral nutrition (EN) through a nasogastric tube. Which findings indicate that the client may be experiencing a complication from the EN? Select all that apply. a.)A weight loss of 2 kg in 24 hours b.)New onset adventitious lung sounds c.)Pale and dry oral mucous membranes d.)Aspirated gastric fluid has a pH of 4 e.) 200 mL dark yellow urine voided in the last eight hours f.)Gastric residual volume of 100 mL

a.)A weight loss of 2 kg in 24 hours b.)New onset adventitious lung sounds c.)Pale and dry oral mucous membranes e.) 200 mL dark yellow urine voided in the last eight hours

The nurse is developing as plan of care for a postoperative client following a radical left mastectomy. Which nursing problem should be the priority for this client? a.)Acute pain related to the surgery b.)Risk of infection of the surgical site c.)Impaired left arm circulation (lymphedema) d.)Anxiety related to the cancer diagnosis

a.)Acute pain related to the surgery

the client was admitted two days ago with a diagnosis of myocardial infarction (MI). When assessing this client, the nurse notes the client's temperature is now 101.1°F (38.5°C). What is the most appropriate nursing intervention? a.)Administer PRN acetaminophen as ordered b.)Increase the client's fluid intake c.)Call the health care provider (HCP) immediately d.)Send blood, urine and sputum for cultures

a.)Administer PRN acetaminophen as ordered

The nurse is caring for a client who was diagnosed with a deep vein thrombosis (DVT). The client reports sudden shortness of breath and the oxygen saturation decreases to 87% on room air. Which intervention is a priority action by the nurse? a.)Administer oxygen to maintain a saturation of 92% b.)Administer the PRN albuterol nebulizer c.)Begin continuous cardiac monitoring d.)Call the health care provider (HCP)

a.)Administer oxygen to maintain a saturation of 92%

The home health nurse is assisting a client who is scheduled for a chemotherapy infusion the next day. The client's medical record indicates frequent episodes of nausea and vomiting after previous chemotherapy treatments. Which action would be most helpful in preventing nausea and vomiting in this client? a.)Administrate metoclopramide prior to start of the infusion. b.)Reinforce teaching to the client about the side effects of chemotherapy. c.)Keep the client on only ice chips for 24 hours after the infusion. d.)Administrate ondansetron immediately after the infusion.

a.)Administrate metoclopramide prior to start of the infusion.

The nurse is providing care for a client who has been diagnosed with terminal cancer. The nurse notes that the client's wife is not visiting very often. When she does visit the client, she only stays for a brief time, stands in the corner and does not approach the client during interactions. Which of the grieving processes is the client's wife most likely experiencing? a.)Anticipatory grief b.)Death anxiety c.)Perceived loss d.)Disenfranchised grief

a.)Anticipatory grief

The nurse is developing a plan of care for an older adult client who will be undergoing a total hip arthroplasty. To improve the client's postoperative recovery, which interventions should the nurse include? Select all that apply. a.)Application of sequential compression device b.)The use of assistive devices for ambulation c.)Preoperative pain control with naproxen d.)Administration of subcutaneous warfarin e.)Instruction on plantar and dorsiflexion exercises

a.)Application of sequential compression device b.)The use of assistive devices for ambulation e.)Instruction on plantar and dorsiflexion exercises

The nurse is caring for a client with dementia who wanders throughout the long-term care facility. Which intervention by the nurse would best ensure the safety of the client? a.)Apply an electronic alert wristband. b.)Explain the risk of walking with no assistance. c.)Reorient the client to time, person and place. d.)Administer an antianxiety medication.

a.)Apply an electronic alert wristband.

The nurse is caring for a client with a dry chest tube drainage system due to a left tension pneumothorax. Two hours ago, the health care provider (HCP) changed the chest tube prescription to water seal only. When entering the client's room, the nurse finds the client to be short of breath, tachypneic and with an oxygen saturation of 84%. On auscultation, the nurse notes absent breath sounds to the left upper lobe. What action should the nurse take first? a.)Apply oxygen via nasal cannula b.)Request a chest X-ray c.)Document all interventions in the client's medical record d.)Notify the appropriate HCP

a.)Apply oxygen via nasal cannula

A client at 39-weeks gestation has just delivered and experienced a fetal demise. The client's partner is at the bedside. Which nursing actions are appropriate at this time? Select all that apply. a.)Ask the parents if they would like to perform any special cultural b.)Clean and wrap the baby and offer it to the parents to view or hold if desired. c.)Stay with the parents and offer supportive care to both of them. d.)Place the infant on the maternal abdomen, skin-to-skin. e.)Offer the option of an autopsy to determine cause of death.

a.)Ask the parents if they would like to perform any special cultural b.)Clean and wrap the baby and offer it to the parents to view or hold if desired. c.)Stay with the parents and offer supportive care to both of them.

The nurse is preparing to administer medications through a gastrostomy tube. The nurse should contact the health care provider before giving which drugs through the gastrostomy tube? Select all that apply. a.)Aspirin EC b.)Calcium carbonate c.)Acetaminophen d.)Diltazem SR e.)Terazosin IR f.)Metoprolol XL

a.)Aspirin EC d.)Diltazem SR f.)Metoprolol XL

The nurse is planning care for a 3-month-old infant in the immediate postoperative period after placement of a ventriculoperitoneal shunt for hydrocephalus. In anticipation of complications of the procedure, the nurse should take which action? a.)Assess for abdominal distention or taut abdominal wall b.)Maintain the infant in supine position c.)Pump the shunt at intervals to assess for proper function d.)Begin formula feedings when infant is alert

a.)Assess for abdominal distention or taut abdominal wall

The nurse is caring for a 78-year-old client with influenza who reports dyspnea on exertion. The client's respirations are shallow and 32 breaths per minute. Which action should the nurse perform first? a.)Auscultate lung fields for breath sounds. b.) Notify respiratory therapy to administer a nebulizer treatment. c.) Apply a non-rebreather mask at 100% oxygen. d.)Evaluate the client's 24-hour intake and output.

a.)Auscultate lung fields for breath sounds.

A client who is two days post abdominal surgery has the following vital signs: blood pressure of 120/70 mm Hg, heart rate of 110 bpm, respiratory rate of 26 breaths per minute and a temperature of 100.4°F (38°C). The client suddenly develops severe shortness of breath, cyanosis and pallor. Which assessment is the priority? a.)Auscultate the lungs for diminished breath sounds. b.)Assess the pupils for unequal responses to light. c.)Palpate the pulses for bounding and irregularity. d.)Check for orthostatic hypotension.

a.)Auscultate the lungs for diminished breath sounds.

The nurse is reinforcing teaching to a 24-year-old woman receiving acyclovir for a Herpes Simplex Virus type 2 infection. Which instructions should the nurse provide the client with? a.)Begin treatment with acyclovir at the onset of symptoms of recurrence b.)Continue to take prophylactic doses for at least five years after the diagnosis c.)Complete the entire course of the medication for an effective cure d.)Stop treatment if she thinks she may be pregnant

a.)Begin treatment with acyclovir at the onset of symptoms of recurrence

The nurse is caring for a child who was diagnosed with coarctation ( narrowing or constriction of the descending aorta, causes increased blood flow to the upper extremities) of the aorta. Which finding should the nurse expect when assessing the child? a.)Bounding pulses in the arms b.)Strong pedal pulses c.)Diminished carotid pulses d.)Normal femoral pulses

a.)Bounding pulses in the arms

The nurse is caring for a client with meningitis. Which observation by the nurse would indicate that the client's condition is worsening? a.)Decreased level of consciousness b.)Complaints of a headache c.)Complaints of light sensitivity d.)A temperature of 101.3° F. (38.5° C)

a.)Decreased level of consciousness

A client is transported to the emergency department with minor injuries suffered during a home fire. The client exhibits intense anxiety after learning that the home was completely destroyed. Which intervention should the nurse implement first? a.)Determine what community and personal support resources are available. b.)Provide a brochure on methods that promote relaxation. c.)Suggest that the client rent an apartment with a sprinkler system. d.)Explore the feelings of grief associated with the loss of the home.

a.)Determine what community and personal support resources are available.

The nurse in a walk-in care clinic is reviewing the medical record of a client who is being treated for frostbite on their toes. Which medical condition most likely placed the client at a higher risk for this type of injury? a.)Diabetes mellitus b.)Hyperthyroidism c.)Systemic lupus erythematosus d.)Aortic stenosis

a.)Diabetes mellitus

The nurse is providing care to an 80-year-old client with the diagnosis of advanced Parkinson's disease. The nurse should know that the greatest risk to the client is related to which finding? a.)Drooling and coughing when eating b.)Dizziness and syncopal episodes c.)Difficulties with reading and seeing at night d.)Extreme weakness in the lower extremities

a.)Drooling and coughing when eating

The nurse is teaching a client about some of the side effects of fluoxetine. What information should the nurse be certain to include? a.)Drowsiness, dry mouth, changes in weight or appetite, reduced libido b.)Orthostatic hypotension, vertigo, hunger, reactions to tyramine-rich foods c.)Tachycardia, blurred vision, hypotension, anorexia d.)Photosensitivity, seizures, edema, hyperglycemia

a.)Drowsiness, dry mouth, changes in weight or appetite, reduced libido

The nurse is reviewing the plan of care for a 30-year-old client newly diagnosed with multiple sclerosis. Which interventions should the nurse include for this client? Select all that apply. a.)Encourage participation in vocational rehabilitation. b.)Instruct the client on how to self-catheterize as needed. c.)Encourage independence in personal care and bathing. d.)Encourage participation in physical and occupational therapy. e.)Review methods to prevent and treat constipation.

a.)Encourage participation in vocational rehabilitation. c.)Encourage independence in personal care and bathing. d.)Encourage participation in physical and occupational therapy. e.)Review methods to prevent and treat constipation.

The nurse is evaluating the plan of care for a client who has been requesting a daily laxative to aid in having a bowel movement. What additional interventions should the nurse include in the client's plan of care? Select all that apply. a.)Encourage the client to drink 2 to 3 liters of fluids a day. b.)Request a prescription for psyllium. c.)Encourage the client to drink more caffeinated beverages. d.)Have the client keep a bowel elimination record. e.)Instruct the client to walk at least 30 minutes 3 to 5 times per week.

a.)Encourage the client to drink 2 to 3 liters of fluids a day. b.)Request a prescription for psyllium. d.)Have the client keep a bowel elimination record. e.)Instruct the client to walk at least 30 minutes 3 to 5 times per week.

The nurse in the primary health care provider's office is speaking with a 40-year-old male client whose most recent hemoglobin A1C level was 9%. The client states that he is motivated to make lifestyle changes to better manage his disease. What interventions should the nurse recommend for this client? Select all that apply. a.)Engage in regular physical activity, such as walking. b.)Schedule an appointment with a registered dietitian. c.)Minimize intake of caffeinated beverages. d.)Check the blood sugar several times a day, ideally before eating. e.)Eliminate all consumption of alcohol. f.)Start a weight loss program until BMI is below 25.

a.)Engage in regular physical activity, such as walking. b.)Schedule an appointment with a registered dietitian. d.)Check the blood sugar several times a day, ideally before eating. f.)Start a weight loss program until BMI is below 25.

A client diagnosed with schizophrenia is observed mumbling to self and speaking to the television. The nursing staff are unable to understand what the client is attempting to communicate. At this time, what is the most desirable outcome for this client's behaviors? a.)Engages in meaningful and understandable verbal communication b.)Expresses feelings appropriately through verbal interactions c.)Demonstrates improved social relationships within the unit d.)Interprets accurately the events and behaviors of others

a.)Engages in meaningful and understandable verbal communication

The nurse is caring for a 60-year-old female client scheduled for abdominal surgery. Which factor in the client's history indicates that the client is at an increased risk for deep vein thrombosis (DVT) in the postoperative period? a.)Estrogen replacement therapy for the past three years b.)History of acute hepatitis A c.)Past hypersensitivity to heparin d.)Family history of uterine cancer

a.)Estrogen replacement therapy for the past three years

The nurse is planning the care for a client who was admitted with complications related to chronic diabetes insipidus. Which interventions are a priority for this client? Select all that apply. a.)Evaluate the moisture level of mucous membranes. b.)Monitor fluid intake and output. c.)Restrict fluid intake to 1000 mL per day. d.)Measure blood glucose levels before meals. e.)Measure blood pressure and heart rate. f.)Weigh the client every morning. g.)Monitor urinary specific gravity.

a.)Evaluate the moisture level of mucous membranes. b.)Monitor fluid intake and output. e.)Measure blood pressure and heart rate. f.)Weigh the client every morning. g.)Monitor urinary specific gravity.

The nurse in the primary health care provider's office is reviewing the medical record of a client with idiopathic pulmonary arterial hypertension. The nurse should expect which potential clinical manifestations with this disease? Select all that apply. a.)Exertional chest pain b.)Abnormal heart sounds c.)Elevated serum creatinine level d.)Dyspnea on exertion e.)Cor pulmonale

a.)Exertional chest pain b.)Abnormal heart sounds d.)Dyspnea on exertion e.)Cor pulmonale

Which assessment findings should make the nurse suspect that the client might have amyotrophic lateral sclerosis (ALS)? a.)Fatigue, progressive muscle weakness and twitching b.)Loss of sensation in the extremities c.)Progressive hearing loss in the last month d.)Complaints of double vision and light sensitivity

a.)Fatigue, progressive muscle weakness and twitching

A client with bipolar disorder is taking lithium. The nurse should notify the health care provider when the client is prescribed which additional medication? a.)Furosemide b.)Amlodipine c.)Finasteride d.)Insulin

a.)Furosemide

The nurse is assigned to care for a client with end-stage liver failure and portal hypertension. Which clinical manifestations would the nurse expect to see with these conditions? Select all that apply. a.)Increased abdominal girth b.)Elevated serum albumin level c.)Diminished pedal pulses d.)Shortness of breath e.)Increased weight gain

a.)Increased abdominal girth d.)Shortness of breath e.)Increased weight gain

A client who is receiving a blood transfusion suddenly reports having a severe headache and low back pain. Which actions should the nurse take? Select all that apply. a.)Obtain a urine specimen as soon as possible. b.)Notify the rapid response team. c.)Complete an incident/occurrence report. d.)Flush the IV line with 30 mL of normal saline. e.)Provide emotional support to the client. f.)Stop the blood transfusion.

a.)Obtain a urine specimen as soon as possible. c.)Complete an incident/occurrence report. e.)Provide emotional support to the client. f.)Stop the blood transfusion.

A client with a history of chronic alcohol use disorder is admitted to the inpatient unit with a serum magnesium level of 1.0 mEq/L. Which intervention should the nurse implement first? a.)Obtain the client's heart rate and oxygen saturation. b.)Assess the client's deep tendon reflexes. c.)Order the client a meal with foods high in magnesium. d.)Place the client on fall risk and seizure precautions.

a.)Obtain the client's heart rate and oxygen saturation.

the nurse is assessing a client who has paraplegia. Which of the following findings would indicate that the client has a probable fecal impaction? a.)Oozing liquid stool b.)Absence of bowel movements c.)Presence of blood in stools d.)Loud continuous flatulence

a.)Oozing liquid stool

The nurse is admitting a 73-year-old client who has a fractured right hip. Which interventions should the nurse include in the client's plan of care? Select all that apply. a.)Palpate the client's bilateral pedal pulses every four hours. b.)Reposition the client every hour to prevent skin breakdown. c.)Ask about the client's pain level with every set of vital signs. d.)Place the client on continuous pulse oximetry. e.)Perform daily circulation, motion and sensation checks on the client's right leg.

a.)Palpate the client's bilateral pedal pulses every four hours. c.)Ask about the client's pain level with every set of vital signs. d.)Place the client on continuous pulse oximetry.

The nurse is caring for a client who has a wound on the leg from a motorcycle accident. During a home visit, the nurse should use which assessment parameter as an indication that this client is experiencing normal wound healing? a.)Pebbled red tissue in the wound base b.)White patches on the outside edges of the wound c.)Eschar over the surface of the wound d.)Green drainage from the center of the wound

a.)Pebbled red tissue in the wound base

The nurse is assessing a client who sustained multiple fractures, contusions, and lacerations in a motor vehicle accident three days ago. The client suddenly becomes confused. Which findings would support the nurse's concern that the client has developed a fat embolism? Select all that apply. a.)Petechiae on the upper anterior chest b.)Low oxygen saturation c.)Elevated temperature d.)Dyspnea Hypertension

a.)Petechiae on the upper anterior chest b.)Low oxygen saturation c.)Elevated temperature d.)Dyspnea

The nurse is admitting a client who does not speak English. Which of the following interventions should the nurse include when caring for the client? Select all that apply. a.)Plan on taking twice as long as usual to complete nursing interventions. b.)Verify that the client's 9-year-old child speaks English well before asking the child to translate. c.)pay attention to any effort by the client to communicate. d.)Speak in a loud voice and use exaggerated hand gestures to communicate with the client. e.)Use a trained medical interpreter provided through the facility's interpreter services. f.) Make a note of the client's preferred language in their medical record.

a.)Plan on taking twice as long as usual to complete nursing interventions. c.) pay attention to any effort by the client to communicate. e.)Use a trained medical interpreter provided through the facility's interpreter services. f.) Make a note of the client's preferred language in their medical record.

The school nurse in an elementary school identifies an outbreak of head lice (pediculosis). Which interventions should the nurse implement to prevent the spread of the infestation? Select all that apply. a.)Provide individual headsets or ear buds for each student. b.)Notify the local health department of the outbreak. c).Do not permit children to share bike helmets. d.)Instruct school parents, teachers and volunteers on how to detect lice and nits. e.)Reassure students that itching of the scalp is a common symptom.

a.)Provide individual headsets or ear buds for each student. c).Do not permit children to share bike helmets. d.)Instruct school parents, teachers and volunteers on how to detect lice and nits. e.)Reassure students that itching of the scalp is a common symptom.

A surgical client with acute pain refuses to participate in physical therapy. The client still has pain despite the administration of pain medication. Based on the information provided, which nonpharmacological intervention(s) would be appropriate for the nurse to add to the plan of care? Select all that apply. a.)Provide the client with a light back massage before physical therapy. b.)Assist the client in meditating before going to physical therapy. c.)Keep the client on strict bedrest until the pain completely resolves. d.)Apply ice directly to the surgical incision 30 minutes before therapy. e.)Ensure the client's room is kept at a comfortable temperature for physical therapy.

a.)Provide the client with a light back massage before physical therapy. b.)Assist the client in meditating before going to physical therapy. e.)Ensure the client's room is kept at a comfortable temperature for physical therapy.

The nurse is caring for a client with end-stage renal disease (ESRD). Which manifestations would the nurse expect to see with this client? Select all that apply. a.)Pruritus b.)Blood pressure of 119/78 c.)Frequent fractures d.)HbA1c of 5.9% e.)Conjunctivitis

a.)Pruritus c.)Frequent fractures e.)Conjunctivitis

The nurse is reviewing the medical record of an inpatient client and notes a positive stool occult blood test. The nurse recognizes which factors may have contributed to this positive result? Select all that apply. a.)Recent invasive dental procedure b.)Amblyopia c.)Current corticosteroid use d.)Current naproxen sodium use e.)Hemiparesis f.)Daily consumption of red meat

a.)Recent invasive dental procedure c.)Current corticosteroid use d.)Current naproxen sodium use f.)Daily consumption of red meat

A client with newly diagnosed irritable bowel syndrome states to the nurse: "All this fiber I have to eat now is making me full of gas! It makes me want to stop taking it." Which actions by the nurse would be appropriate? Select all that apply. a.)Remind client to reduce intake of gas-forming foods b.)Teach client about a balanced and nutrient-rich diet c.)Instruct client to eat only three larger meals every day d.)Instruct client to reduce fiber and then add it again slowly e.)Inquire about a history of lactose intolerance

a.)Remind client to reduce intake of gas-forming foods d.)Instruct client to reduce fiber and then add it again slowly e.)Inquire about a history of lactose intolerance

The nurse is caring for a client with chronic renal failure who is undergoing peritoneal dialysis. The nurse notes that the dialysate solution is instilling very slowly. Which of the following actions would be appropriate for the nurse to implement? Select all that apply. a.)Reposition the client b.)Assess for bruit or vibration c.)Check tubing and catheter for kinks d.)Assess for headache and hypertension

a.)Reposition the client c.)Check tubing and catheter for kinks

The nurse is caring for a client with severe iron deficiency anemia. Which interventions should the nurse include in the client's plan of care? Select all that apply. a.)Review the client's medical record for NSAID use. b.)Monitor the client's stool for color, consistency and frequency. c.)Administer the client's prescribed iron supplements with milk. d.)Instruct assistive personnel to allow the client to rest during care activities. e.)Prepare the client for a packed red blood cells transfusion. f.)Encourage the client to eat more green leafy vegetables and beans. g.)Monitor the client for palpitations and orthostatic hypotension.

a.)Review the client's medical record for NSAID use. b.)Monitor the client's stool for color, consistency and frequency. d.)Instruct assistive personnel to allow the client to rest during care activities. f.)Encourage the client to eat more green leafy vegetables and beans. g.)Monitor the client for palpitations and orthostatic hypotension.

A nurse at a community health clinic is speaking to a group of young adults about preventing HIV infection. Which high risk behaviors to avoid should the nurse include? Select all that apply. a.)Sharing needles b.)Smoking e-cigarettes c.)Inhaling illegal drugs d.)Having unprotected sex

a.)Sharing needles d.)Having unprotected sex

The nurse is caring for a client with late-stage liver cirrhosis. The nurse should monitor the client for which clinical manifestations? Select all that apply. a.)Splenomegaly b.)Increased albumin level c.)Spider angiomas d.)Fluid volume deficit e.)Ascites f.)Encephalopathy g.)Pulmonary hypertension

a.)Splenomegaly c.)Spider angiomas e.)Ascites f.)Encephalopathy

The parent of an 8-month-old infant asks the nurse if the child's language development is appropriate for this age. Which sounds should the nurse anticipate at this age? Select all that apply. a.)Squeals and yells to signal happiness or displeasure b.)Babbles in a rhythm similar to spoken language c.)Meaningful words Single vowel sounds such as ah, eh and uh d.)Vocalizes in response to voices e.)Cooing, gurgling and laughing aloud

a.)Squeals and yells to signal happiness or displeasure c.)Meaningful words Single vowel sounds such as ah, eh and uh d.)Vocalizes in response to voices e.)Cooing, gurgling and laughing aloud

The nurse and client discuss the progress that has been made toward the client's goal of quitting smoking. This is a typical step in which phase of the therapeutic relationship? a.)Termination b.)Orientation c.)Pre-interaction d.)Working

a.)Termination

A nurse is assigned to triage clients who are brought to the emergency department after a bus accident. Which client would be cared for first? a.)The 18-year-old who is diaphoretic and tachycardic and has a board-like abdomen b.)The 9-year-old with dilated pupils and cessation of breathing 10 minutes ago c.)The 35-year-old with an abrasion on the right anterior and lateral side of the chest d.)The 56-year-old with external rotation and shortening of the left lower extremity

a.)The 18-year-old who is diaphoretic and tachycardic and has a board-like abdomen

The nurse is planning the discharge of an 80-year-old female client. Which of the following indicates the client needs to be discharged to a skilled nursing facility instead of home? Select all that apply. a.)The client has a complex surgical dressing change. b.)The client is not able to manage her activities of daily living (ADL). c.)The client is afraid to go home by herself. d.)The client is able to prepare simple meals by herself. e.)The client needs intensive rehabilitation after hip replacement surgery.

a.)The client has a complex surgical dressing change. b.)The client is not able to manage her activities of daily living (ADL). e.)The client needs intensive rehabilitation after hip replacement surgery.

The health care provider (HCP) of a client with opioid-induced constipation prescribed the administration of a bisphosphate enema. After reviewing the client's medical record, the nurse recognizes which contraindications for giving the enema? Select all that apply. a.)The client has a history of thrombocytopenia purpura. b.)The client has a history of hepatitis A. c.)The client has a history of hyperkalemia. d.)The client has a history of hemorrhoidectomy. e.)The client has a history of substance use disorder (SUD). f.)The client has a history of syncopal episodes.

a.)The client has a history of thrombocytopenia purpura. d.)The client has a history of hemorrhoidectomy. f.)The client has a history of syncopal episodes.

The nurse is reviewing the medical record of a client with recurring, nonhealing venous stasis ulcers to the lower extremities. Which findings are most likely contributing to the nonhealing of the client's wounds? Select all that apply. a.)The client smokes one pack of cigarettes per day. b.)The client has a history of seasonal allergies. c.)The client is 74-years-old. d.)The client's ethnicity is Asian American. e.)The client's body mass index (BMI) is 16.5. f.)The client has a history of benign prostatic hyperplasia.

a.)The client smokes one pack of cigarettes per day. c.)The client is 74-years-old. e.)The client's body mass index (BMI) is 16.5.

The nurse in the outpatient clinic is reviewing the medical record of a client diagnosed with Raynaud's disease. What information from the client's health history would support this diagnosis? Select all that apply. a.)The client smokes two packs of cigarettes per day. b.)Warfarin is listed on the medication reconciliation form. c.)Fingers become cyanotic when exposed to cold objects. d.)The client works in an office setting as a typist. e.)The client complains of brittle fingernails that break easily.

a.)The client smokes two packs of cigarettes per day. c.)Fingers become cyanotic when exposed to cold objects. d.)The client works in an office setting as a typist. e.)The client complains of brittle fingernails that break easily.

The nurse is evaluating whether teaching a client with dysphagia about preventing aspiration was effective. Which action by the client indicates that additional teaching is required? a.)The client uses a straw to drink. b.)The client tucks in the chin while swallowing. c.)The client is sitting in a chair during meals. d.)The client alternates solids with liquids.

a.)The client uses a straw to drink.

A nurse working on the orthopedic unit has just received change-of-shift report. Which client should the nurse evaluate first? a.)The client who has not voided 10 hours after a laminectomy. b.)The client who is anxious about discharge to a rehabilitation facility. c.)The client with low back pain rated 8 out of 10. d.)The client with osteomyelitis and a temperature of 100.5°F.

a.)The client who has not voided 10 hours after a laminectomy.

A client is seen at the primary care clinic for allergic rhinitis. Which clinical manifestations should the nurse expect with this diagnosis? Select all that apply. a.)Watery, itchy, reddened eyes b.)Alteration in sense of smell c.)Purulent, green nasal discharge d.)Increase in serum eosinophil count e.)Worsening of symptoms during spring and fall season

a.)Watery, itchy, reddened eyes b.)Alteration in sense of smell d.)Increase in serum eosinophil count e.)Worsening of symptoms during spring and fall season

The nurse is preparing to administer a feeding through a percutaneous endoscopic gastrostomy tube. What nursing action is needed before starting the feeding? Select all that apply. a..)Verify the length and placement of the tube b.)Palpate the abdomen c.)Keep the feeding product refrigerated until ready to use d.)Milk or massage the tube e.)Elevate the head of the bed 30 to 45 degrees f.)Flush the tube with 30 mL of warm water

a..)Verify the length and placement of the tube e.)Elevate the head of the bed 30 to 45 degrees f.)Flush the tube with 30 mL of warm water

The nurse is providing discharge education to a client hospitalized for an acute exacerbation of rheumatoid arthritis. The nurse includes information focusing on conserving energy. Which statements by the client demonstrate the teaching has been effective? Select all that apply. a.) "On days of increased pain, I will stay in bed and relax instead of being active." b.) "If possible, I will delegate some things to my friends or family." c.) "I will sit on a tall kitchen stool instead of standing when I am preparing meals." d.) "I will schedule activities throughout the day instead of trying to complete everything in the morning." e.) "I will set priorities and complete the important tasks first."

b.) "If possible, I will delegate some things to my friends or family." c.) "I will sit on a tall kitchen stool instead of standing when I am preparing meals." d.) "I will schedule activities throughout the day instead of trying to complete everything in the morning." e.) "I will set priorities and complete the important tasks first."

A woman who is 15 weeks pregnant verbalizes concern to the nurse about weight gain during pregnancy. Which statement indicates a correct understanding of weight changes for a woman during the second trimester? a.) "Don't worry about weight gain. You are eating for two." b.) "You should gain about one pound each week." c.) "Try to gain as little as possible during this trimester." d.) "Expect to gain between 3 to 5 pounds during this trimester."

b.) "You should gain about one pound each week."

The hospital has sounded the call for a disaster drill on the evening shift. Which client would be most appropriate to be discharged to make room for a new admission? a.) A client with a history of lupus who was admitted with Stevens-Johnson syndrome that morning b.) A client with a history of being ventilator-dependent who was admitted with pneumonia eight days ago c.) A client with type 2 diabetes who was admitted for acute cellulitis of the lower leg 48 hours ago d.)A client newly diagnosed with type 1 diabetes who was admitted with antibiotic-induced diarrhea 24 hours ago

b.) A client with a history of being ventilator-dependent who was admitted with pneumonia eight days ago

The hospital nurse on the evening shift of a busy telemetry unit is asked to determine which assigned client is a candidate for discharge. Which client should the nurse select as the best candidate for discharge? a.) A client who is actively dying and has a "do not resuscitate" order. b.) A client with a history of type 1 diabetes and resolving diabetic ketoacidosis. c.) A client admitted at the beginning of the shift with an exacerbation of asthma. d.) A client admitted to rule out acute pancreatitis with a current blood sugar of 90 mg/dL.

b.) A client with a history of type 1 diabetes and resolving diabetic ketoacidosis.

The nurse is evaluating a toddler's readiness for toilet training. Which milestones should the nurse assess to determine the readiness of the child? Select all that apply. a.) Ability to button clothing when dressing themselves b.) Ability to have a dry diaper for two hours and wake from a nap with a dry diaper c.)Recognition of the urge to defecate or urinate d.)Parents willing to invest the time needed to teach the child e.)Fine motor development enough to be able to remove clothing

b.) Ability to have a dry diaper for two hours and wake from a nap with a dry diaper c.)Recognition of the urge to defecate or urinate d.)Parents willing to invest the time needed to teach the child e.)Fine motor development enough to be able to remove clothing

The nurse is teaching a client who has coronary artery disease about nutrition. What information should the nurse include? a.) Limit sodium intake to 7 g per day. b.) Avoid large and heavy meals. c.) Eat foods rich in vitamin K. d.) Do not exceed 40 grams of carbohydrates a day.

b.) Avoid large and heavy meals.

The nurse working in an intensive care unit is caring for a client diagnosed with acute angina. The client is receiving an intravenous infusion of nitroglycerin. What is the priority assessment for this client? a.) Urine output b.) Blood pressure c. Heart rate d.) Neurologic status

b.) Blood pressure

The nurse is reviewing the client's medical record and notes that the client has been taking an oral contraceptive for several years. For which potential complications should the nurse monitor the client? Select all that apply. a.) Osteoporosis b.) Breast cancer c.) Deep Vein Thrombosis (DVT) d.) Colon cancer e.) Depression f.)Anemia

b.) Breast cancer c.) Deep Vein Thrombosis (DVT) e.) Depression

A child is treated with succimer for lead poisoning. Which of these assessments is the priority? a.) Check the client's blood calcium level. b.) Check the client's complete blood count with differential. c.) Check the client's serum potassium level. d.) Test the client's deep tendon reflexes.

b.) Check the client's complete blood count with differential.

The nurse is caring for a client who has cystic fibrosis. The nurse would expect the client to be prescribed which type of diet? a.) Gluten-free, low fiber b.) High fat, high-calorie c.) Sodium-restricted d.) Dairy-free

b.) High fat, high-calorie

The nurse receives an order to administer intravenous gentamicin to a client. For which finding should the nurse contact the health care provider to clarify the order? a.) Low serum blood urea nitrogen b.) High serum creatinine c.)Low serum albumin d.) High gastric pH

b.) High serum creatinine

The home health nurse is performing a daily dressing change on a client who has a diabetic ulcer. Which intervention is most important for the nurse to implement to meet the goal of wound healing? a.) Evaluate the client's understanding of appropriate foot care. b.) Involve the client in making heath care decisions. c.) Schedule regular visits to monitor wound healing. d.) Arrange for a referral to a diabetic educator.

b.) Involve the client in making heath care decisions.

An 80-year-old client arrives in the emergency room after a fall at home. The client has several large skin abrasions. Which action should the nurse perform first? a.) Clean and apply an appropriate dressing to the abrasions. b.) Perform a head-to-toe assessment. c.) Document findings of alterations in the skin's integrity. d.) Verify if the client has advance directives in place.

b.) Perform a head-to-toe assessment.

A client who has been receiving chemotherapy through a central venous access device (CVAD) at home, is admitted to the intensive care unit with a diagnosis of septicemia. Which nursing intervention is the priority? a.) Place the client on contact precautions. b.) Prepare the client for insertion of a new CVAD. c.) Change the dressing over the site of the existing CVAD. d.) Insert an indwelling urinary catheter.

b.) Prepare the client for insertion of a new CVAD.

A community health clinic nurse is interviewing a client who is experiencing lightheadedness. The client reports a history of arthritis and is taking ibuprofen for the pain. The client is pale with a blood pressure of 88/40 mmHg. The client's pulse is 114 bpm, the respiratory rate is 22 breaths per minute, and the oral temperature is 98.2°F (36.7 °C). Which additional information should the nurse solicit from the client? Select all that apply. a.) Presence of tingling or numbness in the extremities b.) Presence of bruises c.) Frequency and amount of ibuprofen used d.) Presence of photophobia e.) Color of bowel movements

b.) Presence of bruises c.) Frequency and amount of ibuprofen used e.) Color of bowel movements

The nurse is planning care for several clients in the labor and delivery unit. Which mother-baby pair would the nurse identify as needing a Coombs test? a.)Rh-positive mother with Rh-negative baby b.) Rh-negative mother with Rh-positive baby c.) Rh-negative mother with Rh-negative baby d.) Rh-positive mother with Rh-positive baby

b.) Rh-negative mother with Rh-positive baby

A nurse is reviewing laboratory results for a client diagnosed with acute renal failure. Which result should be reported to the primary health care provider immediately? a.) Venous blood pH of 7.30 b.) Serum potassium of 6 mEq/L c.) Blood urea nitrogen of 50 mg/dL d.) Hemoglobin of 9.3 g/dL

b.) Serum potassium of 6 mEq/L

The community health nurse is planning a teaching session for a family with children about safety and risk-reduction in their home. What information is most important to obtain prior to the session to ensure the teaching is effective? a.) The ages and occupations of the parents. b.) The ages of the children in the home. c.) The number of children in the home. d.) The physical layout of the home.

b.) The ages of the children in the home.

a client is transported to the emergency department after a motor vehicle accident. When assessing the client 30 minutes after arrival, the nurse notes several physical changes. Which finding requires immediate attention? a.) Tachycardia b.) Tracheal deviation c.) Increased restlessness d.) Tachypnea

b.) Tracheal deviation

The nurse is caring for a client diagnosed with gastroenteritis, caused by a Salmonella infection. Which intervention should the nurse implement to prevent transmission of this infection? a.) Wear two pairs of gloves when changing linens. b.) Wash hands with soap and water after client contact. c.) Isolate the client in a single room without a roommate. d.) Place the client on contact precautions.

b.) Wash hands with soap and water after client contact.

The nurse is caring for a 30-year-old female client scheduled for a hypophysectomy due to a pituitary tumor. The client asks how removal of the pituitary gland will affect her Which is the best response by the nurse? a.)"Because of the surgery, you will be at an increased risk for seizures." b.)"After the procedure, you might have difficulties getting pregnant." c.)"You will be required to monitor your blood sugar levels and take insulin." d.)"You will be immunocompromised and need to stay away from large crowds."

b.)"After the procedure, you might have difficulties getting pregnant."

The nurse in the neurology office is reviewing information about levetiracetam with a 30-year-old female client with a history of seizures. Which instruction about the medication should the nurse make sure to include? a.)"You should avoid becoming pregnant while taking this medication." b.)"Call the office immediately if you feel like hurting or killing yourself." c.)"You should stay away from large crowds and sick children." d.)"You might experience irregular menses and intermittent bleeding."

b.)"Call the office immediately if you feel like hurting or killing yourself."

The nurse is collecting data from an adolescent client. Which of the following issues should the nurse address? Select all that apply. a.)"Have you decided what you are going to do after high school?" b.)"How are things going at home?" c.)"Have you gotten in any trouble lately?" d.)"Are you currently having conflicts with someone close to you?" e.)"Where are you currently living?" f.)"How many sexual partners have you had in the past six months?"

b.)"How are things going at home?" d.)"Are you currently having conflicts with someone close to you?" e.)"Where are you currently living?" f.)"How many sexual partners have you had in the past six months?"

The nurse understands that the prescribed levothyroxine is effective when the client with hypothyroidism makes which statement? a.)"I still feel lethargic and fatigued." b.)"I have been having daily, formed bowel movements." c.)"I was reprimanded at work after becoming angry with my boss." d.)"I have to change my sheets in the morning because I sweat a lot at night."

b.)"I have been having daily, formed bowel movements."

A client presents with elevations in triiodothyronine (T3) and thyroxine (T4) and with normal thyroid-stimulating hormone (TSH) levels. Which is the nurse's priority intervention? a.)Check for Trousseau's sign. b.)Monitor the apical pulse. c.)Administer levothyroxine. d.)Administer propranolol.

b.)Monitor the apical pulse.

The home health nurse is reviewing the medical record of a client with closed-angle glaucoma in both eyes. Which statement by the client would support this diagnosis? a.)"I can't see out of my left eye." b.)"I have to turn my head to see around the room." c.)"I have constant blurred vision." d.)"I have specks floating in my eyes."

b.)"I have to turn my head to see around the room."

The nurse is teaching a client how to properly use their peak flow meter at home. Which statement by the client indicates an understanding of the teaching? a.)"I will exhale completely then inhale quickly into the mouthpiece." b.)"I will take a deep breath and then blow out hard and fast into the mouthpiece." c.)"I will exhale completely then inhale slowly and steadily into the mouthpiece." d.)"I will take a deep breath and exhale slowly and steadily into the mouthpiece."

b.)"I will take a deep breath and then blow out hard and fast into the mouthpiece."

The nurse in the primary care office is following up with a client who has been experiencing frequent constipation. Which statement by the client about using psyllium (Metamucil) indicates that additional teaching is needed? a.)"I may notice some bloating while I am taking it." b.)"I will take it together with my other medications." c.)"I will only take it until my constipation is relieved." d.)"I will take it in the morning with lots of water."

b.)"I will take it together with my other medications."

The health care provider writes a new order for a fentanyl patch to manage chronic pain experienced by a client in hospice care. The nurse is teaching the client and family members about the fentanyl patch and knows that teaching was effective when the client makes which of the following statements? Select all that apply. a.)"I should cut up the patch before I throw it away so no one else can use it." b.)"It may take up to a half day or longer for the patch to start working, the first time I use it." c.)"I will take the old patch off before I apply the new patch on." d.)"I can soak in a hot tub to help decrease my pain." e.)"If my pain is too great while I am on the patch, I can take a supplemental pain medication."

b.)"It may take up to a half day or longer for the patch to start working, the first time I use it." c.)"I will take the old patch off before I apply the new patch on." e.)"If my pain is too great while I am on the patch, I can take a supplemental pain medication."

The nurse is caring for a group of adult clients on a neurological unit in an acute care hospital. Which client should the nurse see first? a.)A client admitted with hepatic encephalopathy who has an elevated ammonia level b.)A client admitted several hours ago with a subdural hematoma due to an unwitnessed fall at home c.)A client admitted with a transient ischemic attack, who has a bubble study echocardiogram ordered d.)A client admitted two days ago with an ischemic stroke who has a blood pressure of 158/64

b.)A client admitted several hours ago with a subdural hematoma due to an unwitnessed fall at home

The nurse is reviewing various group activities with the health care team. When planning a therapeutic milieu, what is the most important factor when selecting a group activity? a.)Provide consistency with clients' skills b.)Achieve clients' therapeutic goals c.)Raise the level of group participation d.)Match it to the clients' preferences

b.)Achieve clients' therapeutic goals

A client with a history of asthma and kidney stones is admitted with a diagnosis of recurrent renal calculi. The client experiences shortness of breath following a lithotripsy. The nurse auscultates the client's lungs and finds decreased air movement with no wheezing. The arterial blood gas (ABG) results are pH 7.31, PaO2 53 mm Hg, PaCO2 50 mm Hg, and O2 sat 82%. Which of the following actions are appropriate for the nurse to take? Select all that apply. a.)Start oxygen via nasal cannula b.)Administer a short-acting bronchodilator via nebulizer c.)Start high flow oxygen via face mask d.)Call respiratory therapy e.)Contact the health care provider f.)Prepare for possible intubation g.)Increase IV fluids

b.)Administer a short-acting bronchodilator via nebulizer c.)Start high flow oxygen via face mask d.)Call respiratory therapy e.)Contact the health care provider f.)Prepare for possible intubation

A client diagnosed with an acute anterior myocardial infarction is receiving nitroglycerin and heparin intravenously. The client still reports chest pain. Which action should the nurse take? a.)Administer antidysrhythmic drugs as indicated. b.)Administer intravenous morphine sulfate as ordered. c.)Auscultate heart and lung sounds. d.)Review and compare serial ECG strips.

b.)Administer intravenous morphine sulfate as ordered.

The nurse is caring for a client newly diagnosed with generalized anxiety disorder (GAD) who has been prescribed alprazolam by the health care provider (HCP). Which of the following statements best describes this medication in the treatment of GAD? a.)Alprazolam will become more effective over time. b.)Alprazolam provides short-term treatment but is less effective than other drug therapy. c.)There is no risk for developing a dependency to alprazolam. d.)Alprazolam is the only recommended drug for GAD.

b.)Alprazolam provides short-term treatment but is less effective than other drug therapy.

The nurse is caring for a client in the intensive care unit who is experiencing shock. The client's blood pressure has decreased from 128/74 to 100/40 mm Hg, within one hour. Which compensatory physiologic response should the nurse expect to see? a.)An increase in urine output b.)An increase in the heart rate c.)A decrease in level of consciousness d.)A decrease in the respiratory rate

b.)An increase in the heart rate

A 67-year-old client is admitted to the telemetry unit with substernal chest pressure that radiates to the jaw. The client's diagnosis is an acute myocardial infarction. To monitor the client, the nurse should give priority to which assessment? a.)Assess the client's level of anxiety. b.)Assess the client's cardiac output. c.)Assess the client's pain level. d.)Assess the client's activity tolerance.

b.)Assess the client's cardiac output.

The nurse is caring for a client who is experiencing urinary incontinence. Which of the following teaching points should the nurse reinforce when discussing this health issue with the client? a.)Avoid eating foods high in sodium b.)Avoid taking antihistamines c.)Hold voiding or urination d.)Restrict fluids

b.)Avoid taking antihistamines

The nurse is caring for a 4-year-old child admitted after being burned over more than 50% of the body. Which of the following laboratory data should be reviewed by the nurse as a priority in the initial 24 hours after the burn? a.)Hematocrit b.)Blood urea nitrogen c.)White blood count d.)Blood glucose

b.)Blood urea nitrogen

The nurse is assessing a client with a hip fracture who has been in Buck's traction for 24 hours. Which nursing assessment is the priority? a.)Auscultating the client's lung sounds. b.)Checking the pulse, temperature and sensation of the client's lower extremities. c.)Monitoring the client's skin for breakdown around bony prominences. d.)Assessing the client's level of pain.

b.)Checking the pulse, temperature and sensation of the client's lower extremities.

A client is diagnosed with amyotrophic lateral sclerosis (ALS). Which nursing action will help prevent the complications of atelectasis and pneumonia in this client? a.)Repositioning every two hours around the clock b.)Chest physiotherapy twice a day c.)Active and passive range of motion exercises twice a day d.)Use of the incentive spirometer every two hours while awake

b.)Chest physiotherapy twice a day

The nurse is providing postoperative care for a client following a laparoscopic cholecystectomy. Which assessment finding should be of highest concern? a.)Client has absent bowel sounds. b.)Client reports right upper quadrant pain. c.)Client is drowsy. d.)Client reports shoulder discomfort.

b.)Client reports right upper quadrant pain.

The nurse is reviewing the medical record of a client who is scheduled for a computerized tomography (CT) scan of the brain with contrast. For which information in the client's medical record should the nurse notify the health care provider? Select all that apply. a.)Client has a mechanical heart valve. b.)Client takes anticonvulsant medication. c.)Client has an iodine allergy d.)Client has a positive pregnancy test. e.)Client has peripheral neuropathy. f.)Client is on hemodialysis.

b.)Client takes anticonvulsant medication. c.)Client has an iodine allergy d.)Client has a positive pregnancy test. f.)Client is on hemodialysis.

A client is admitted for hypovolemia associated with multiple draining wounds. Which is the best method for the nurse to use to evaluate the client's fluid balance? a.)Presence of edema b.)Daily weight c.)Hourly urine output d.)Skin turgor

b.)Daily weight

The nurse is reviewing the medical record of a client admitted with acute kidney injury. Which findings would support this diagnosis? Select all that apply. a.)Decreased blood area nitrogen b.)Decreased glomerular filtration rate c.)Hypokalemia d.)Hematuria e.)Elevated creatinine level f.)Proteinuria

b.)Decreased glomerular filtration rate d.)Hematuria e.)Elevated creatinine level f.)Proteinuria

The nurse is caring for a client with Type I diabetes. Which finding requires immediate intervention by the nurse? c.)Intense thirst and increased urination a.)Reduced sensation in the periphery b.)Diaphoresis and shakiness d.)Mild discomfort at the injection site

b.)Diaphoresis and shakiness

A client is admitted to the hospital after falling at home and is given an opioid analgesic for pain. Two days later, the client develops delirium. What clinical manifestation(s) should the nurse monitor the client for? Select all that apply. a.)Gaps in long-term memory b.)Disturbances in sleep-wake pattern c.)Confusion that will last several months d.)Rambling, incoherent speech e.)Fluctuating emotions

b.)Disturbances in sleep-wake pattern d.)Rambling, incoherent speech e.)Fluctuating emotions

The nurse is caring for a client who was admitted to the hospital for syndrome of inappropriate antidiuretic hormone. Which interventions are appropriate for this client's plan of care? Select all that apply. a.)Increase the client's fluid intake to 5,000 milliliters per day. b.)Document the client's weight daily. c.)Monitor the client for pulmonary edema and orthopnea. d.)Administer five units of vasopressin every six hours, intramuscularly. e.)Keep a padded tongue blade at the bedside. f.)Monitor the client's serum sodium level. g.)Document changes in the client's neurologic status.

b.)Document the client's weight daily. c.)Monitor the client for pulmonary edema and orthopnea. f.)Monitor the client's serum sodium level. g.)Document changes in the client's neurologic status.

The nurse in a long-term care facility is preparing to administer medications. Which physiological changes does the nurse know will affect medication pharmacokinetics in older adults? a.)Due to an increase in metabolism, medications are prescribed more frequently. b.)Due to a decrease in renal drug excretion, a greater risk for adverse medication effects exists. c.)Due to a decrease in gastric emptying, higher medication doses are prescribed. d.) to an increase in glomerular filtration rates, medications are excreted more rapidly.

b.)Due to a decrease in renal drug excretion, a greater risk for adverse medication effects exists.

The nurse is reviewing the electronic medical record of a client diagnosed with endometriosis. The nurse should expect which findings with this diagnosis? Select all that apply. a.)Amenorrhea b.)Dysmenorrhea c.)Dyspareunia d.)Infertility e.)Urinary tract infection

b.)Dysmenorrhea c.)Dyspareunia d.)Infertility

The nurse is working with a group of clients who have eating disorders. The clients are to have their weight taken wearing only a hospital gown and underwear. No street clothing is allowed. Which is the most appropriate explanation for this practice? a.)Symbolically removes barriers between the client and staff b.)Eliminates the risk of hiding objects in clothing or shoes c.)Promotes feelings of success with gaining weight d.)Allows the nurse to better assess the client's skin

b.)Eliminates the risk of hiding objects in clothing or shoes

The nurse is working at an adult day care program. An older adult client who has been diagnosed with a neurocognitive disorder (dementia) is crying and repeatedly saying, "I want to go home. Call my mommy to come get me." Which intervention by the nurse is most appropriate? a.)Give the client detailed information about what the client will be doing throughout the day. b.)Engage with the client and prompt the client to join an exercise group. c.)Inform the client that they must wait until the program ends at 5 pm to leave. d.)Direct the client to a group activity while reorienting them to person and place.

b.)Engage with the client and prompt the client to join an exercise group.

The nurse is reviewing the documentation of a client's care in their electronic health record and realizes that one of the entries was completed on the wrong client. Which actions are appropriate for the nurse to take? Select all that apply. a.) Delete the entry from the record. b.)Mark the entry as "Mistaken entry. Wrong client." c.)Complete an incident report. d.)Notify the nurse manager. e.)Enter the time the error was discovered. f.)Notify the health care provider.

b.)Mark the entry as "Mistaken entry. Wrong client." e.)Enter the time the error was discovered.

The nurse is planning care for a client newly diagnosed with essential hypertension. Which interventions should the nurse include in the client's plan of care? Select all that apply. a.)Encourage the client to limit smoking to one pack of cigarettes per day. b.)Evaluate the client's ability to take their own blood pressure. c.)Encourage the client to take daily, 30-minute walks. d.)Explain the negative effects of hypertension on the body. e.)Evaluate the client's understanding of a low-sodium diet. f.)Instruct the client to abstain from drinking any alcohol.

b.)Evaluate the client's ability to take their own blood pressure. c.)Encourage the client to take daily, 30-minute walks. d.)Explain the negative effects of hypertension on the body. e.)Evaluate the client's understanding of a low-sodium diet.

the nurse is caring for a client with a calcium imbalance related to hypoparathyroidism. The nurse should anticipate which clinical manifestation in this client? a.)Decreased neuromuscular excitability b.)Facial twitching when the region over cranial nerve VII is tapped c.)Decreased gastrointestinal activity and constipation d.)Bounding peripheral pulses in the lower extremities

b.)Facial twitching when the region over cranial nerve VII is tapped

The nurse is caring for a client with anemia of chronic disease. The client's latest hemoglobin level is 7.6 g/dL. Which clinical manifestations would the nurse expect to find? Select all that apply. a.)Hypertension b.)Fatigue c.)Tachypnea d.)Pallor e.)Bradycardia

b.)Fatigue c.)Tachypnea d.)Pallor

The nurse is caring for a client who fell two hours ago while alone in their room. The client appears tired and disoriented. What should the nurse do first? a.)Ensure fall precautions are included in the client's plan of care. b.)Gather data about the client's baseline neurologic status. c.)Use a gait belt to assist the client to their bed for a nap. d.)Reorient the client to their surroundings, date and time.

b.)Gather data about the client's baseline neurologic status.

The nurse is caring for a 12-year-old client with thalassemia. What lab value is most important to monitor for this client? a.)Prothrombin time b.)Hemoglobin level c.)Serum creatinine level d.)Platelet count

b.)Hemoglobin level

A client is admitted to the hospital with endocarditis. The nurse understands that which risk factors can lead to the development of endocarditis? (Select all that apply.) a.)Treatment of substance use disorder with methadone b.)History of aortic valve replacement c.)Placement of a central venous access device d.)Oral abscess with tooth extraction e.)Placement of an arteriovenous fistula for hemodialysis f.)Atrial fibrillation with use of warfarin

b.)History of aortic valve replacement c.)Placement of a central venous access device d.)Oral abscess with tooth extraction e.)Placement of an arteriovenous fistula for hemodialysis

A hospitalized 8-month-old infant is receiving digoxin to treat Tetralogy of Fallot. Prior to administering the next dose of the medication, the parent reports that the baby vomited one time, just after breakfast. The infant's heart rate is 92 bpm. What action should the nurse take? a.)Double the next dose to make up for the medication lost from vomiting. b.)Hold the medication and notify the primary health care provider. c.)Give the scheduled dose after the client is done eating lunch. d.)Reduce the next dose by half and then resume the normal medication schedule.

b.)Hold the medication and notify the primary health care provider.

A home health nurse is teaching the parents of a pediatric client with acute spasmodic croup. Which interventions are most important to include? a.)Antihistamines to decrease allergic responses b.)Humidified air with an increase in oral fluids c.)Sedation as needed to prevent exhaustion d.)Antibiotic therapy for 10 to 14 days

b.)Humidified air with an increase in oral fluids

The nurse is reviewing the medical record of a client who has been hospitalized for a pulmonary embolism 3 times in the past six months. The nurse understands that which potential intervention is appropriate for this client? a.)Hemodialysis b.)Inferior vena cava filter c.)Lung resection surgery d.)Prophylactic alteplase infusion

b.)Inferior vena cava filter

The nurse is providing care to an older adult client diagnosed with bilateral pneumonia. Which intervention should the nurse implement to best promote the client's comfort? a.)Increase the client's oral fluid intake. b.)Keep conversations short. c.)Monitor vital signs frequently. d.)Encourage visits from family and friends.

b.)Keep conversations short.

The nurse in an emergency department is assessing a client who fell at home 24 hours ago. Which finding requires the nurse's immediate attention? a.)Heart rate of 98 bpm b.)Large bruise behind one ear c.)Baseline blood pressure of 150/90 mm Hg d.)Atrial fibrillation on the ECG monitor

b.)Large bruise behind one ear

The nurse is talking with a client with schizophrenia when the client abruptly says, "The moon is full. Astronauts walk on the moon. Walking is a good health habit." Which is the best description for the client's speech pattern? a.)Neologisms b.)Loose associations c.)Word salad d.)Flight of ideas

b.)Loose associations

The nurse is administering medication to a client who does not speak English. Which of the following strategies should the nurse implement to ensure the client understands the purpose of the medication? Select all that apply. a.)Use correct medical terminology during instructions. b.)Maintain eye contact with the client, even when speaking to an interpreter. c.)Use the translation phone line to interpret information between the client and nurse. d.)Communicate through a facility-approved interpreter. e.)Plan to take a longer amount of time than usual for medication administration.

b.)Maintain eye contact with the client, even when speaking to an interpreter. c.)Use the translation phone line to interpret information between the client and nurse. d.)Communicate through a facility-approved interpreter. e.)Plan to take a longer amount of time than usual for medication administration.

The nurse is caring for a client with bronchiolitis. What is the priority nursing intervention? a.)Administering 100% oxygen. b.)Maintaining a patent airway. c.)Maintaining a soft diet. d.)Administering antiviral medications.

b.)Maintaining a patent airway.

The nurse is caring for a client who was admitted with a diagnosis of renal calculi. The client reports moderate-to-severe flank pain and nausea. The client's oral temperature is 100.8 °F (38.2 °C). Which goal is the priority for this client? a.)Prevent infection b.)Manage pain c.)Control nausea d.)Maintain fluid balance

b.)Manage pain

The nurse suspects cardiac tamponade in a client who has acute pericarditis. How should the nurse determine the presence of pulsus paradoxus? a.)Check the electrocardiogram for dysrhythmias during the respiratory cycle. b.)Note when Korotkoff sounds are auscultated during inspiration and expiration. c.)Listen for a pericardial friction rub when the client is instructed to hold their breath. d.)Subtract the diastolic blood pressure from the systolic blood pressure.

b.)Note when Korotkoff sounds are auscultated during inspiration and expiration.

The nurse on a postpartum nursing unit is receiving report about a client who had a normal spontaneous vaginal delivery the night before. The client has been passing golf ball-sized clots on her peri-pad for the last few hours. The client's most recent blood pressure is 88/56 mm Hg, and her heart rate is 118 bpm. The nurse enters the client's room and notices blood oozing from her intravenous insertion site. Which action should the nurse take first? a.)Encourage breastfeeding to promote uterine contractions. b.)Notify the client's health care provider. c.)Palpate and massage the client's uterus. d.)Perform peri-care and change the client's peri-pad.

b.)Notify the client's health care provider.

A client being treated for hypertension and depression tells the nurse at the health clinic, "I recently purchased a handgun because I am thinking about suicide." Which action should the nurse take first? a.)Phone the family to warn them of the risk b.)Notify the health care provider immediately c.)Complete the physical and mental health assessment d.)Suggest inpatient psychiatric care

b.)Notify the health care provider immediately

The school nurse is called to see a student who is having a nosebleed that will not stop. The student's records show a history of hemophilia. What are the priority interventions for this client? Select all that apply. a.)Excuse the student from all future physical exercise classes. b.)Notify the student's health care provider. c.)Apply direct pressure by squeezing the nose tightly. d.)Notify the student's parent or guardian(s). e..)Notify the school's principal. f.)Remind the student to avoid blowing the nose too forcefully.

b.)Notify the student's health care provider. c.)Apply direct pressure by squeezing the nose tightly. d.)Notify the student's parent or guardian(s). f.)Remind the student to avoid blowing the nose too forcefully.

A newly admitted client reports taking phenytoin for several months. Which assessment should the nurse include in the admission report? Select all that apply. a.)Report of anorexia, numbness and tingling of the extremities b.)Report of unsteady gait, rash and diplopia c.)Serum phenytoin levels d.)Report of any seizure activity

b.)Report of unsteady gait, rash and diplopia c.)Serum phenytoin levels d.)Report of any seizure activity

A 48-year-old male client who is being admitted to the emergency department with an acute myocardial infarction (MI) gives the following list of medications to the nurse. Which medication would the nurse recognize as having the most immediate implications for the client's care? a.)Losartan b.)Sildenafil c.)Furosemide d.)Captopril

b.)Sildenafil

A nurse in a rural community uses telehealth to provide care and education to clients in remote locations. What are the perceived benefits of using telehealth? a.)Telehealth standardizes electronic sharing of health information data. b.)Telehealth removes the time and distance barriers from the delivery of care. c.)Telehealth greatly reduces health care costs for the clients who use it. d.)Telehealth empowers clients to take a greater interest in their illness.

b.)Telehealth removes the time and distance barriers from the delivery of care.

The nurse is reviewing the plan of care for a client with a sacral, stage III pressure ulcer who is prescribed continuous negative-pressure wound therapy (NPWT). For which finding should the nurse notify the health care provider (HCP) immediately? a.)The wound has extensive tunneling. b.)The client is receiving apixaban. c.)The client is incontinent of stool. d.)The client is receiving enteral nutrition.

b.)The client is receiving apixaban.

The home health nurse is caring for a client who underwent a partial gastrectomy due to gastric cancer several months ago. Which finding would indicate that the client is suffering from pernicious anemia? Select all that apply. a.)The client is exhibiting alopecia. b.)The client reports numbness and tingling in the feet. c.)The client's tongue is shiny and beefy-red. d.)The client is experiencing urinary retention. The client's sclerae are icteric.

b.)The client reports numbness and tingling in the feet. c.)The client's tongue is shiny and beefy-red. e.)The client's sclerae are icteric.

Which information in a client's history would place them at an increased risk for skin cancer? Select all that apply. a.)The client is dark-skinned. b.)The client's profession is fisherman. c.)The client is 65-years-old. d.)The client has blond hair and green eyes. e.)The client is receiving an immunosuppressant drug.

b.)The client's profession is fisherman. c.)The client is 65-years-old. d.)The client has blond hair and green eyes. e.)The client is receiving an immunosuppressant drug.

the nurse is reviewing medication orders for a client who has requested something for pain. In the process, the nurse finds a new written order for a pain medication. The health care provider wrote, "Give APAP every six hours as needed for pain." Which parts of the medication order should the nurse clarify before administering the medication? Select all that apply. a.)The mechanism of action b.)The indication c.)The route d.)The dosage e.)The frequency f.)The drug name

b.)The indication c.)The route d.)The dosage f.)The drug name

The nurse is preparing to enter a disaster scene to assist with triaging victims. What assessment priorities should the nurse adhere to? Select all that apply. a.) The nurse must consult a qualified health care provider prior to making client resource decisions. b.)The nurse should assess clients by considering their airway, breathing, circulation and neurological function. c.)The nurse requires disaster certification before performing triage during a disaster. d.)The nurse should consider the age of a victim before allocating any resources. e.)The nurse should allocate resources to those victims with the strongest probability of survival.

b.)The nurse should assess clients by considering their airway, breathing, circulation and neurological function. e.)The nurse should allocate resources to those victims with the strongest probability of survival.

The nurse is caring for a client who is receiving regular insulin, supplied in a glass vial. Which step(s) should the nurse take to ensure the correct administration of the insulin? Select all that apply. a.)The nurse should store unopened vials of insulin in the freezer. b.)The nurse should check the strength of the insulin before administering it. c.)The nurse should store opened vials of insulin at room temperature. d.)The nurse should only use an insulin syringe to administer insulin. e.)The nurse should shake the insulin vial before drawing up the insulin. f.)The nurse should rub the injection site after administering the insulin. g.)The nurse should discard the vial 28 days after it was opened

b.)The nurse should check the strength of the insulin before administering it. c.)The nurse should store opened vials of insulin at room temperature. d.)The nurse should only use an insulin syringe to administer insulin. g.)The nurse should discard the vial 28 days after it was opened

A client diagnosed with testicular cancer has undergone a unilateral orchiectomy. The client expresses fears about his prognosis. What should the nurse understand about this type of cancer? a.)Intensive chemotherapy is the treatment of choice following surgery. b.)This cancer has a five-year survival rate of 90% or greater with early diagnosis and treatment. c.)With early intervention, the cure rate for testicular cancer is about 50%. d.)This surgery causes impotence and infertility.

b.)This cancer has a five-year survival rate of 90% or greater with early diagnosis and treatment.

The nurse in a long-term care facility is reviewing the medical record of a newly admitted client. Which of the following factors put the client at an increased risk for developing a pressure ulcer? Select all that apply. a.) the client is alert and oriented to person, place, time and situation. b.The client has diabetes mellitus. c.)The client has a body mass index (BMI) of 30. d.)The client is receiving an immunosuppressant drug for rheumatoid arthritis. e.)The client has a history of exercise-induced asthma.

b.The client has diabetes mellitus. c.)The client has a body mass index (BMI) of 30. d.)The client is receiving an immunosuppressant drug for rheumatoid arthritis.

The nurse is educating a client concerning the risk factors of osteoporosis. Which statement by the client would indicate additional education is needed? a.) "Weight-bearing exercises may help increase my bone density." Incorrect b.) "My female gender puts me at higher risk for osteoporosis." c.) "A few cups of coffee a day will not increase my risk of osteoporosis." d.) "Some common medications have been linked to an increased risk of osteoporosis."

c.) "A few cups of coffee a day will not increase my risk of osteoporosis."

The nurse is providing information to a client about propranolol. Which statement by the client indicates the teaching has been effective? a.) "I could have an increase in my heart rate for a few weeks." b.) "I should expect to feel nervousness during the first few weeks." c.) "I can have a heart attack if I stop this medication suddenly." d.) "I may experience seizures if I stop the medication abruptly."

c.) "I can have a heart attack if I stop this medication suddenly."

A client who is scheduled for a diagnostic mammography asks the nurse about the cancer risk from radiation exposure. Which response is most appropriate by the nurse? a.) "This exam does not use radiation, and it is not dangerous." b.) "A chest X-ray gives you more radiation exposure." c.) "The radiation from mammography is equivalent to one hour of sun exposure." d.) "You have nothing to worry about; it is less than tanning in the nude."

c.) "The radiation from mammography is equivalent to one hour of sun exposure."

An emergency room nurse is assigned to the triage area of a nearby mass casualty event. Which of these clients should the nurse tag as "Black" or "to be seen last"? a.) An 83-year-old client with an open fracture of the left arm b.) A 7-month-old infant with closed fractures to both lower legs who is crying loudly c.) A 45-year-old client with second and third degree burns over 90% of their body d.) A 14-year-old client with a small amount of bright red blood dripping from their nose

c.) A 45-year-old client with second and third degree burns over 90% of their body

1. The nurse receives their client care assignment for the upcoming shift. The nurse has been assigned a client with a nephrostomy tube. The nurse has never cared for a client with a nephrostomy tube before. Which is the most appropriate action by the nurse? a.) Conduct a literature review about proper nephrostomy tube care. b.) Plan to check-in with the charge nurse and the client often during the upcoming shift. c.) Ask the charge nurse to change the assignment to a different nurse. d.) Ask the charge nurse to provide an in-service about nephrostomy tube care.

c.) Ask the charge nurse to change the assignment to a different nurse.

The nurse is caring for a client with a nasogastric tube and is preparing to administer an enteral feeding through the tube. Which is the best method to confirm correct tube placement prior to beginning the feeding? a.) Auscultate the abdomen while instilling 10 mL of air into the tube. b.) Measure the length of tubing from nose to epigastrium. c.) Check the pH level of the aspirated contents. d.) Place the end of the tube in water to check for air bubbles.

c.) Check the pH level of the aspirated contents.

The nurse enters a toddler's hospital room to administer an oral medication. When the nurse asks the child, "Are you ready to take your medicine?" the child's response is an immediate, "No!" Which action would be appropriate by the nurse? a.) Ask another nurse to hold the child while giving the medication. b.) Notify the health care provider and request a parenteral form of the medication. c.) Leave the room and return five minutes later to try to give the medication. d.) Explain to the child that the medicine must be taken now.

c.) Leave the room and return five minutes later to try to give the medication.

A client with schizophrenia is admitted to a mental health center with acute paranoia. The client tells the nurse: "I am a government official being followed by spies." Upon further questioning, the client states: "My warnings must be heeded to prevent nuclear war." Which action is most appropriate for the nurse to take? b.) Contact the government agency. c.) Listen quietly without comments. a.) Ask for more information about the spies. d.) Confront the client's delusions.

c.) Listen quietly without comments.

The nurse is assessing a client in her third trimester of pregnancy. The recent ultrasound suggests the baby is small for the gestational age. However, an earlier ultrasound indicated normal growth. The nurse understands that this change is most likely associated with which problem? a.) Exposure to teratogens b.) Chromosomal abnormalities c.) Maternal hypertension d.) Sexually transmitted infection

c.) Maternal hypertension

The nurse is caring for an 80-year-old client who requires wrist restraints. Which client behaviors would support the need to continue to use restraints? Select all that apply. a.) The client is pushing the call light button every 10 minutes. b.) The client is directing frequent angry, verbal outbursts at the staff. c.) The client is confused and trying to pull out an IV catheter. d.)The client is resisting care and attempting to hit the staff. e.) The client is trying to get out of bed without assistance.

c.) The client is confused and trying to pull out an IV catheter. d.)The client is resisting care and attempting to hit the staff.

the nurse is teaching a 65-year-old female client who is newly diagnosed with osteoporosis. Which type of exercise is best for this client? a.)"Do yoga to strengthen muscles and protect bones." b.)"Start a weight loss program to reduce your weight." c.)"Do weight-bearing exercise or resistance activities." d.)"Go running 3 to 5 times per week."

c.)"Do weight-bearing exercise or resistance activities."

The nurse is caring for a client who was recently diagnosed with hypopituitarism. Which client statements would indicate additional teaching is needed? Select all that apply. a.)"I should let my health care provider know if there are changes in my urinary patterns." b.)"I should feel less fatigued within the next few weeks." c.)"I should expect breast swelling or tenderness." d.)"I should expect to feel more thirsty throughout the day." e.)"I will need to take calcium and vitamin D supplements."

c.)"I should expect breast swelling or tenderness." d.)"I should expect to feel more thirsty throughout the day."

The nurse is caring for a client admitted with a phosphorus level of 1.5 mg/dL. Which statement by the client should alert the nurse to collect further data about possible causes for the phosphate imbalance? Select all that apply. a.)"I had my parathyroid gland surgically removed." b.)"I have a history of decreased kidney function." c.)"I take a calcium supplement with every meal." d.)"I do not eat any meat or dairy products." e.)"I snack on nuts in-between meals."

c.)"I take a calcium supplement with every meal." d.)"I do not eat any meat or dairy products."

The nurse is reinforcing teaching for a client with Bell's palsy. Which statement by the client indicates that additional teaching is needed? a.)"I will apply warm, moist heat to my face." b.)"I will eat and drink using the strong side of my mouth." c.)"I will rest my facial muscles because they are recovering." d.)"I will wear an eye patch at night while I sleep."

c.)"I will rest my facial muscles because they are recovering."

The nurse is working with a client who is diagnosed with multiple sclerosis on how to reduce muscle spasticity. Which statement by the client indicates the need for further teaching? a.)"If I exercise daily, it can help with relieving spasticity." b.)"My stretching routine can help with the muscle spasms." c.)"Taking a long hot bath may relieve the muscle spasms." d.)"Taking the prescribed muscle relaxant will relieve painful spasms."

c.)"Taking a long hot bath may relieve the muscle spasms."

During the well-baby check-up of a 3-week-old newborn, the parent expresses concern to the nurse because a flat pink birthmark on the baby's forehead and eyelid has not gone away. What is the appropriate response by the nurse? a.)"This birthmark is called seborrheic keratoses and could indicate a cancerous growth." b.)"Birthmarks are generally harmless and will disappear as the baby grows." c.)"This birthmark is called a port-wine stain and can be associated with other malformations." d.)"This birthmark happened during birth and may require surgical removal at a later time."

c.)"This birthmark is called a port-wine stain and can be associated with other malformations."

A client calls the clinic and states to the triage nurse: "I had an upset stomach and took Pepto-Bismol and now my tongue looks black. What's happening to me?" What would be the nurse's best response? a.)"Come to the clinic so you can be seen by the health care provider." b.)"Are your stools also black?" c.)"This is a common and temporary side effect of this medication." d.)"How long have you had an upset stomach?"

c.)"This is a common and temporary side effect of this medication."

A client experienced a myocardial infarction and is preparing for discharge from the hospital. The client asks the nurse what activity will indicate it is safe to resume sexual intercourse. Which instruction will the nurse provide? a.)"Have a glass of wine to relax you, then you can try to have sex." b.)"If you can maintain an active walking program, you will have less risk." c.)"When you can climb one to two flights of stairs without problems, it is generally safe." d.)"You must regain all your strength before attempting such exertion."

c.)"When you can climb one to two flights of stairs without problems, it is generally safe."

A 2-year-old child is brought to the pediatrician's office by the parents, who report that the child has been having diarrhea for two days. What nutritional information should the nurse provide to the parents? a.)Give the child bananas, apples, rice and toast as tolerated. b.)Keep the child fasting, give them nothing to eat, and return the next day. c.)Continue a regular diet and add electrolyte replacement drinks. d.)Give the child only clear liquids and gelatin for 24 hours.

c.)Continue a regular diet and add electrolyte replacement drinks.

The nurse in a pediatrician's office is speaking with the parent of an 8-year-old child who is concerned about the child receiving the annual flu vaccine due to an egg allergy. How should the nurse respond? a.)"We have new types of flu vaccines where an egg allergy does not matter." b.)"We can premedicate the child to prevent an allergic reaction." c.)"You can schedule an appointment to have the vaccine administered in our office." d.)"Your child should not be receiving the flu vaccine."

c.)"You can schedule an appointment to have the vaccine administered in our office."

The nurse from a women's wellness health clinic is temporarily assigned to an adult medical unit. Which of these client assignments would be most appropriate for this nurse? a.)A newly diagnosed client with type 2 diabetes mellitus who is learning about foot care b.)A newly admitted client with a diagnosis of pancreatic cancer and severe dehydration c.)A client who was in a motor vehicle accident who has an external fixation device on their leg d.)A client admitted for a barium swallow after a transient ischemic attack.

c.)A client who was in a motor vehicle accident who has an external fixation device on their leg

The health care provider orders blood tests for a client diagnosed with acute hepatitis B (HBV). Which lab finding should the nurse anticipate to be elevated? a.)BUN (blood urea nitrogen) b.)Albumin c.)ALT (alanine aminotransferase) d.)WBC (white blood cells)

c.)ALT (alanine aminotransferase)

A nurse is administering the influenza vaccine in an occupational health clinic. Within 10 minutes of giving the vaccine to a middle-aged adult male, the man reports having itchy and watery eyes, feeling anxious and short of breath. What should the nurse do first? a.)Apply oxygen. b.)Take the client's vital signs. c.)Administer SQ epinephrine. d.)Maintain the airway.

c.)Administer SQ epinephrine.

The nurse is reviewing the medical record of a client with a new prescription for lovastatin for hyperlipidemia. Which finding requires the nurse to notify the health care provider immediately? a.)Serum creatinine level of 1.2 mg/dL b.)Hemoglobin A1c level of 10.2% c.)Alanine aminotransferase level of 90 U/L d.)Total cholesterol level of 320 mg/dL

c.)Alanine aminotransferase level of 90 U/L

The nurse is caring for a 14-year old child who has been tentatively diagnosed with hyperthyroidism. Which of these findings noted on the initial nursing assessment require intervention by the nurse? a.)A report of irritability worsening in the past two weeks b.)A comment by the client: "I just can't sit still." c.)An apical heart rate of 190 bpm d.)A 10% weight loss in the last month, despite an excellent appetite

c.)An apical heart rate of 190 bpm

Which condition should the nurse correlate with the following arterial blood gas values: pH 7.48, HCO3 22 mEq/L, PCO2 28 mm Hg, PO2 98 mm Hg? a.)Chronic obstructive pulmonary disease b.)Diabetic ketoacidosis c.)Anxiety-induced hyperventilation d.)Diarrhea and vomiting for 36 hours

c.)Anxiety-induced hyperventilation

A nurse is caring for a trauma victim who experienced significant blood loss. The client has received multiple transfusions. Which test would be the most accurate indicator of oxygenation? a.)Hemoglobin and hematocrit (H and H) b.)Complete blood count (CBC) c.)Arterial blood gases (ABGs) d.)Pulse oximetry

c.)Arterial blood gases (ABGs)

A client is admitted to the psychiatric unit after a suicide attempt. Which of the following interventions is important for the nurse to implement initially? Select all that apply. a.)Help the client identify the stressors that precipitated their current crisis b.)Ask why the client attempted suicide c.)Ask the client directly if they have suicidal thoughts or plan to commit suicide d.)Establish a trusting, therapeutic relationship e.)Identify resources that the client may use after discharge f.)Assign a staff member to stay with the client at all times

c.)Ask the client directly if they have suicidal thoughts or plan to commit suicide d.)Establish a trusting, therapeutic relationship f.)Assign a staff member to stay with the client at all times

A client is admitted for first and second degree burns on the face, neck, anterior chest and hands. Which action should be the nurse's priority? a.)Initiate intravenous therapy. b.)Administer pain medication. c.)Assess for dyspnea or stridor. d.)Cover the areas with dry sterile dressings.

c.)Assess for dyspnea or stridor.

he nurse in a long-term care facility is reviewing the plan of care for a female client diagnosed with a urinary tract infection. To reduce the risk of recurrence, which interventions should the nurse include in the plan of care? Select all that apply. a.)Bathe the client rather than have her shower. b.)Provide the client with at least 1 liter of water a day. c.)Assist the client with wiping the perineum front to back. d.)Have the client void every 2 to 3 hours. e.)Discourage the client from drinking coffee or black tea.

c.)Assist the client with wiping the perineum front to back. d.)Have the client void every 2 to 3 hours. e.)Discourage the client from drinking coffee or black tea.

The nurse is assessing a client admitted for acute exacerbation of chronic obstructive pulmonary disease. Which assessment finding would support this diagnosis? a.)Crackles in the lung bases b.)Inspiratory laryngeal stridor c.)Audible expiratory wheezing d.)An S3 heart sound

c.)Audible expiratory wheezing

A new nurse is delegating tasks to the unlicensed assistive personnel (UAP). If delegated, which task would require intervention by the nurse manager? a.)Feed a 2-year-old with a broken arm b.)Empty the urethral collection bag and provide perineal care c.)Bathe a woman receiving brachytherapy with an internal radon device d.)Assist an elderly client to the restroom

c.)Bathe a woman receiving brachytherapy with an internal radon device

Which finding should alert the nurse to the possible presence of a cataract in a client? a.)Nearsightedness and loss of peripheral vision b.)Farsightedness and loss of central vision c.)Blurred vision and reduced color perception d.)Dull aching in the eye and eyelids

c.)Blurred vision and reduced color perception

Two members of the interdisciplinary team are arguing about the plan of care for a client. Which strategy could be used to de-escalate the situation? a.)Tell the team members they must calm down and be reasonable. b.)Interrupt, apologize for the interruption and change the subject. c.)Bring the communication focus back to the client. d.)Adjourn the meeting and reschedule when everyone has calmed down.

c.)Bring the communication focus back to the client.

The nurse in the outpatient clinic is reviewing discharge instructions with a client being treated for recurring sinusitis. Which statement by the client indicates that additional teaching is needed? Select all that apply. a.)Restrict water intake to reduce copious nasal drainage. b.)Reduce the use of smokeless tobacco and cigarettes. c.)Control asthma symptoms with prescribed medications. d.)Sleep in a flat position to decrease postnasal drip. e.)Wash hands and change clothing after outdoor activities. f.)Use nasal decongestant sprays several times daily. g.)Use probiotics daily to reduce recurrence.

c.)Control asthma symptoms with prescribed medications. e.)Wash hands and change clothing after outdoor activities. g.)Use probiotics daily to reduce recurrence.

The nurse is caring for a child diagnosed with Kawasaki disease. The nurse should monitor the child for which potential complication? a.)Occlusions at the vessel bifurcations b.)Pulmonary embolism c.)Coronary artery aneurysm d.)Chronic vessel plaque formation

c.)Coronary artery aneurysm

the pediatric nurse is screening a child for suspected lead poisoning. Which assessment finding would support this diagnosis? a.)Obesity b.)Excessive perspiration c.)Developmental delays d.)Enuresis

c.)Developmental delays

A client is admitted to the medical-surgical unit following a motor vehicle accident. Twelve hours after admission, the client becomes diaphoretic, tremulous and irritable, and the client's pulse and blood pressure are elevated. The client states to the nurse, "I have to get out of here." Which is the most likely cause for the client's symptoms and behavior? a.)Dissatisfaction with hospital care b.)Anxiety related to being hospitalized c.)Early stage of alcohol withdrawal d.)Shock related to the injuries

c.)Early stage of alcohol withdrawal

The nurse is caring for a client who received tenecteplase to open an occluded coronary artery. Which finding should be of highest concern for the nurse? a.)Urinary retention b.)Bleeding gums c.)Hematemesis d.)Epistaxis

c.)Hematemesis

The nurse is admitting a client newly diagnosed with hypertension. Which of the following is the best method for assessing the client's blood pressure? a.)With arms hanging down b.)With legs crossed c.)In both arms d.)After exercising

c.)In both arms

A client is admitted with a tentative diagnosis of left-sided heart failure. Which assessment finding is consistent with this diagnosis? a.)Chest pain b.)Heart murmur c.)Inspiratory crackles d.)Cyanosis

c.)Inspiratory crackles

The oncology nurse is caring for a female client who is being treated for metastatic breast cancer. The client is scheduled to receive their first dose of trastuzumab. Which assessment finding is most important to notify the health care provider of? a.)Absolute neutrophil count 2.5 (2,500 mm3) b.)Intermittent nausea and vomiting c.)Irregular apical pulse d.)Blood glucose 130 mg/dL

c.)Irregular apical pulse

Several hours after a total gastrectomy, the client's nasogastric tube (NGT) stops draining. After referring to the postoperative orders, which order will the nurse implement first? a.)Increase the amount of suction by 5 mmHg b.)Reposition the tube until it begins to drain c.)Irrigate the nasogastric tube d.)Notify the surgeon

c.)Irrigate the nasogastric tube

A nurse working in a nursing home is caring for an older adult client who has been diagnosed with a urinary tract infection. Which finding should be of greatest concern to the nurse? a.)Suprapubic pain b.)Cloudy urine c.)Low blood pressure d.)Confusion

c.)Low blood pressure

The nurse administers cimetidine to a 75-year-old client diagnosed with a gastric ulcer. The nurse should monitor the client for which adverse reaction? a.)Hearing loss b.)Increased liver enzymes c.)Mental status change d.)Constipation

c.)Mental status change

The nurse is caring for a client who is prescribed erythromycin 500 mg orally every six hours for the treatment of pneumonia. The nurse should monitor the client for which common side effect? a.)Esophagitis b.)Orange-red discoloration of urine c.)Nausea and vomiting d.)Tendon rupture

c.)Nausea and vomiting

The nurse is caring for a client who received thrombolytic therapy for an acute myocardial infarction (MI). Which information is most important for the nurse to communicate to the health care provider (HCP)? a.)A decrease in ST-segment elevation on the ECG b.)A large bruise at the client's IV insertion site c.)No change in the client's reported level of chest pain d.)An increase in troponin levels from baseline

c.)No change in the client's reported level of chest pain

The nurse is assigned to a client who is newly diagnosed with active tuberculosis. Which intervention is the priority? a.)Collect several sputum samples for testing. b.)Reinforce hand washing before and after entering the room. c.)Place the client in a private, negative pressure room. d.)Have the client dispose of soiled tissues in a separate bag.

c.)Place the client in a private, negative pressure room.

A client received hydromorphone orally one hour ago. When the nurse enters the client's room, the client is unresponsive to verbal stimuli and has a respiratory rate of six breaths per minute. Which action should the nurse take next? a.)Begin cardiopulmonary resuscitation. b.)Administer supplemental oxygen. c.)Prepare to administer naloxone. d.)Prepare for endotracheal intubation.

c.)Prepare to administer naloxone.

The home health nurse is developing a plan of care for a 3-year-old client diagnosed with cerebral palsy (CP). Which goals are the priority for this client? Select all that apply. a.)Select appropriate school environment b.)Arrange for genetic counseling c.)Promote locomotion d.)Treat muscle spasms e.)Prevent seizures

c.)Promote locomotion d.)Treat muscle spasms e.)Prevent seizures

The nurse is caring for a client who was seriously injured in a bus accident. Several people were killed in the accident, including the client's son. The client's spouse, who was not injured, has had frequent outbursts of yelling at the staff. The client's spouse is now threatening legal action due to "inadequate care." Which interventions should the nurse implement? Select all that apply. a.) Notify the health care provider about the situation. b.) Notify hospital security to remove the client's spouse. c.)Provide information about grief support groups. d.)Request a change in client assignment. e.)Allow the spouse to express their feelings.

c.)Provide information about grief support groups. e.)Allow the spouse to express their feelings.

The nurse in the emergency department is admitting a client with a reduced level of consciousness due to severe hypothyroidism. Which intervention should the nurse implement first? a.)Orient the patient to person, time and place. b.)Administer propranolol as prescribed. c.)Provide supplemental oxygen. d.)Implement warming blankets as indicated.

c.)Provide supplemental oxygen.

The nurse is assessing a 1-day-old newborn infant. The nurse notices that the infant's breasts are enlarged bilaterally with a thin, white discharge. Which action by the nurse is appropriate? a.)Notify the health care provider immediately. b.)Ask the mother about medications taken during pregnancy. c.)Record the findings, noting they are normal. d.)Obtain a specimen of the fluid to check for glucose.

c.)Record the findings, noting they are normal.

A client received 40 mg of furosemide by mouth at 10 am. Which information is most important for the nurse to provide to the next nurse in the change-of-shift report? a.)The client lost two pounds in the last 24 hours. b.)The client is to receive another dose of furosemide at 10 pm. c.)The client's urine output was 1500 mL over nine hours. d.)The client's potassium level was 4.0 mEq/L prior to administration.

c.)The client's urine output was 1500 mL over nine hours.

An 18-month-old child is awaiting a renal transplant. When reviewing the child's health history, the nurse notes that the child has not had the first measles, mumps, rubella (MMR) immunization. Which action should the nurse take? a.)Live vaccines are withheld in children with renal chronic illness. b.)The risk of the vaccine's side effects are too great and it should not be given. c.)The vaccine should be given now, before the transplant. d.)An inactivated form of the vaccine can be given at any time

c.)The vaccine should be given now, before the transplant.

a client is about to undergo a plaster cast application. Prior to the cast application, the nurse should include what teaching point in the discussion? a.)The cast material will be dipped several times into the tepid water b.)The cast should be covered with cotton material until it fully dries c.)The wet cast should be handled with the palms of hands until fully dry d.)The casted extremity will be placed on a cloth-covered surface

c.)The wet cast should be handled with the palms of hands until fully dry

The nurse is caring for a client diagnosed with testicular cancer. Which risk factor supports this diagnosis? a.)Genital herpes b.)Older than 60 years of age c.)Undescended testis d.)Benign prostatic hyperplasia

c.)Undescended testis

The nurse is providing care for a client who is diagnosed with schizophrenia and treated with clozapine. The client reports that his leg has developed an involuntary movement and he can feel his heart beating. Which other assessment findings should the nurse gather before calling the health care provider (HCP)? a.)Bowel sounds in all four abdominal quadrants b.)Total urinary output for the last 24 hours c.)Vital signs including oral temperature d.)Glasgow Coma Scale (GCS) to measure level of consciousness

c.)Vital signs including oral temperature

The nurse is counseling a postpartum client who has a history of a substance-abuse problem. Which question is a priority when interviewing the client? a.)Do you feel that you have bonded with your infant? b.)How have you managed the stress of being a new mother? c.)When was the last time you used illegal substances? d.)Have you attended any support groups related to substance abuse?

c.)When was the last time you used illegal substances?

The nurse is caring for a client who is receiving isoniazid for tuberculosis (TB). Which assessment finding would indicate the client is having a possible adverse response to this medication? a.)Tinnitus and decreased hearing b.)Headache and nausea c.)Yellowing of the sclera d.)Tingling in extremities

c.)Yellowing of the sclera

The nurse is examining a 2-year-old child with a tentative diagnosis of Wilm's tumor. Which statement by the child's parent should the nurse follow-up on? a.) "All of my child's pants have become tight around the waist." b.) "My child prefers some salty foods more than others." c.) "My child has lost 3 pounds in the last month." d.) "My child seems to be urinating less over the past 2 days."

d.) "My child seems to be urinating less over the past 2 days."

The nurse is caring for a male client admitted with a diagnosis of a spinal cord injury. The client asks the nurse how the injury will affect his ability to have sex. Which is the best response by the nurse? a.) "Normal sexual function is not possible." b.) "Sexual functioning will not be impaired at all." c.) "There are drugs to help with achieving an erection." d.) "Sexual intercourse may be possible."

d.) "Sexual intercourse may be possible."

A 3-year-old client has just returned from surgery for application of a hip spica cast. Which nursing action should the nurse implement? a.) Use the crossbar to help turn the child from side to side. b.) Position the child flat in the bed and reposition from supine to prone every 2-4 hours. c.) Drying the cast using a hair dryer set to "warm". d.) Apply waterproof plastic tape to the cast around the genital area.

d.) Apply waterproof plastic tape to the cast around the genital area.

The nurse is teaching a 9-year-old child to self-administer bronchodilators for asthma management. Which cognitive developmental milestone should the nurse consider for a child of this age? a.) Children of this age are egocentric and highly imaginative. b.) Abstract logic is useful for teaching concepts to children this age. c.) Children of this age apply concepts from one context to another. d.) Children of this age can think logically in the organization of facts.

d.) Children of this age can think logically in the organization of facts.

The nurse in a pediatric office is speaking with the parents of a 3-year-old child. The parents ask the nurse about normal growth and development. The nurse knows that which finding may be indicative of abnormal childhood development? a.) Presence of a pincer grasp b.) Presence of all primary teeth c.) Use of four-word sentences d.) Positive Babinski reflex

d.) Positive Babinski reflex

The nurse is caring for a nullipara client who, at 12-weeks gestation is beginning prenatal care. The client has just learned she is positive for human immunodeficiency virus (HIV). Which of the following statements by the nurse is important for the client to understand regarding infection prevention for her baby? a.)"Pregnancy is known to accelerate the course of your illness." b.)"Breastfeeding is recommended because the health benefits outweigh the risks of HIV transmission" c.)"Medication for HIV will be started immediately after birth for both you and your baby." d.)"A cesarean section will be scheduled before your membranes rupture."

d.)"A cesarean section will be scheduled before your membranes rupture."

The registered nurse (RN) is giving instructions to an unlicensed assistive personnel (UAP) regarding care activities for a new admission . Which directive provides the best information about assigned tasks? a.)"Let me know how the new admission is doing and tell me if you need any help." b.)"You will need to frequently take an oral temperature for the client and report the results to me immediately if it is too high." c.)"Before 12 pm today, ambulate the client and replace the sequential compression device (SCD) afterwards." d.)"Beginning at 8 am, empty the urinary catheter bag hourly and write the amount and time on the whiteboard."

d.)"Beginning at 8 am, empty the urinary catheter bag hourly and write the amount and time on the whiteboard."

The nurse is assigned to care for a client diagnosed with HIV/AIDS. A first-semester nursing student asks the nurse how a diagnosis of AIDS is determined, other than a positive HIV test. What response by the nurse is the best explanation for how AIDS is diagnosed? a.)"Having a CD4+ lymphocyte count less than 400 and a positive Western Blot test." b.)"Having symptoms of anxiety, dementia, depression and insomnia, along with a low viral load." c.)"The presence of any number of opportunistic infections and testing positive on the viral load test." d.)"Having a CD4+ lymphocyte count less than 200 and one or more AIDS defining conditions."

d.)"Having a CD4+ lymphocyte count less than 200 and one or more AIDS defining conditions."

The nurse is providing discharge education to a client diagnosed with coronary artery disease. The client is prescribed to use a nitroglycerin transdermal patch at home. Which statement by the client indicates a correct understanding of safe medication administration? a.)"I can place this patch on broken skin. It will absorb better." b.)"I will keep a record of chest pain occurrences now that I have this patch." c.)"This drug can lead to hypertension. So, I will monitor my blood pressure at home." d.)"I will remove the old patch and cleanse the area before applying a new patch."

d.)"I will remove the old patch and cleanse the area before applying a new patch."

The nurse is evaluating a client post kidney transplant about the client's understanding of mycophenolate mofetil. Which statement by the client indicates a need for further teaching? a.)"I will notify my doctor when I develop a sore throat and chills." b.)"I will take Tylenol for minor aches and pains." c.)"I will take the medication on an empty stomach." d.)"I will take milk of magnesia with it to prevent heartburn."

d.)"I will take milk of magnesia with it to prevent heartburn."

the nurse is teaching a client with rheumatoid arthritis about etanercept. Which of the following statements by the client indicates no further teaching is needed? a.)"The medication needs to be mixed well. You can shake the bottle to mix it." b.)"Take the medication daily, first thing in the morning on an empty stomach." c.)"You will need to come into the clinic every 6 weeks to receive an intravenous infusion." d.)"If you keep the medication in a refrigerator, be sure to allow it to warm to room temperature before injecting it."

d.)"If you keep the medication in a refrigerator, be sure to allow it to warm to room temperature before injecting it."

A male client who is diagnosed with gonococcal urethritis tells the nurse he had recent sexual contact with a woman who did not appear to have any disease. What is the best response by the nurse? a.)"Women might not have the disease but can be a carrier and infect others." b.)"Gonorrhea in women only affects the ovaries and not the genital organs." c.)"Men are at a much greater risk than women for acquiring gonorrhea." d.)"Women might not realize that they have gonorrhea because they are often asymptomatic."

d.)"Women might not realize that they have gonorrhea because they are often asymptomatic."

A client who had a wrist cast applied three days ago calls from home, reporting that the cast is loose enough to slide off. How should the nurse respond? a.)"Place several gauze bandages inside the cast to prevent it from sliding off." b.)"Use an arm sling to keep the casted arm immobile." c.)"As your muscles atrophy, the cast is expected to loosen." d.)"You need a new cast now that the swelling is decreased."

d.)"You need a new cast now that the swelling is decreased."

After receiving report on the following clients, which client should the nurse assess first? a,)A client diagnosed with emphysema with questions about a new medication b.)A client who underwent a partial gastrectomy and reports feeling lightheaded c.)A client reporting gastric distress after taking ibuprofen d.)A client diagnosed with peptic ulcer disease (PUD) who reports feeling dizzy

d.)A client diagnosed with peptic ulcer disease (PUD) who reports feeling dizzy

To which nursing home resident could a nurse safely administer tricyclic antidepressants (TCAs) without questioning the health care provider's order? a.)A client with narrow-angle glaucoma b.)A client with benign prostatic hypertrophy (BPH) c.)A client with coronary artery disease (CAD) d.)A client with mild hypertension

d.)A client with mild hypertension

Which lab result would the nurse expect to find that is consistent with a diagnosis of hemophilia A? a.)A deficiency of white blood cells b.)A deficiency of red blood cells c.)A deficiency of clotting factor IX d.)A deficiency of clotting factor VIII

d.)A deficiency of clotting factor VIII

The nurse is providing care to an adult client in the postanesthesia care unit who is recovering from an emergency appendectomy. The client is sleepy, but arousable and reports zero pain on a numeric pain scale. Which assessment finding requires immediate action by the nurse? a.)A pulse oximetry reading of 92% b.)A temperature of 99.1°F (37.3°C ) c.)A blood pressure of 100/60 mmHg d.)A resting heart rate of 128 bpm

d.)A resting heart rate of 128 bpm

A client is admitted to the telemetry unit with syncope due to sinus bradycardia. Which intervention should the nurse include in the client's plan of care? a.)Maintain the client on bedrest. b.)Discuss the client's wishes for organ donation. c.)Implement seizure precautions. d.)Administer a stool softener daily.

d.)Administer a stool softener daily.

The nurse is providing care for a 40-year-old client suspected of having Guillain-Barré syndrome. Which intervention should the nurse plan for? a.)A bone marrow biopsy b.)Genetic testing of the client's children c.)Implementation of airborne precautions d.)Administration of immunoglobulins

d.)Administration of immunoglobulins

The nurse is monitoring the level of consciousness for a client who experienced a head injury. During the last assessment, the client scored a 15 on the Glasgow Coma Scale (GCS). Now, the client opens eyes to verbal command (GCS 3), has purposeful movement to painful stimulus (GCS 5) and is using inappropriate words (GCS 3). Which intervention by the nurse should be implemented first? a.)Continue to monitor level of consciousness b.)Increase the flow of oxygen c.)Raise the head of the bed d.)Call the rapid response team and health care provider

d.)Call the rapid response team and health care provider

The nurse is teaching a group of adolescents about sexually-transmitted infections. Which should the nurse emphasize as the most common sexually-transmitted infection? a.)Gonorrhea b.)Human immunodeficiency virus (HIV) c.)Herpes d.)Chlamydia

d.)Chlamydia

The nurse is teaching a client how to collect a clean catch urine specimen. What is the appropriate sequence? a.)Void a little, clean the meatus, then collect specimen b.)Void into the toilet, then collect specimen from the toilet c.)Clean the meatus, then urinate into the container d.)Clean the meatus, begin voiding, then collect sample midstream

d.)Clean the meatus, begin voiding, then collect sample midstream

The nurse is completing a health history of a client diagnosed with Alzheimer's disease. The nurse reviews a list of the client's medications and supplements routinely taken at home. Which treatment should be a cause for concern by the nurse? a.) Omega-3 fatty acids b.) Ginkgo biloba c.)Donepezil d.)Coconut oil

d.)Coconut oil

The nurse is performing a surgical dressing change on a client who had a laparotomy five days ago. The nurse notices the incision edges are separated and there is a visible bulge of organ tissue protruding from the wound opening. Which is the best way for the nurse to dress the incision before leaving the room to call the surgeon? a.)Apply antibiotic ointment to the wound and cover it with a non-adherent dressing. b.)Place iodine-soaked gauze over the wound and then cover it with an abdominal pad. c.)Approximate the wound edges as much as possible with wound-closure strips. d.)Cover the wound with sterile gauze moistened with sterile 0.9% saline.

d.)Cover the wound with sterile gauze moistened with sterile 0.9% saline.

The nurse is caring for a child who is receiving treatment for lead poisoning. The nurse understands that the most serious effect of lead exposure is related to which of the following? a.)impaired kidney function b.)Lead colic and constipation c.)Anemia and fatigue d.)Damage to the central nervous system

d.)Damage to the central nervous system

A client with a history of bipolar disorder is admitted to the hospital after a suicide attempt. Which of the following interventions should the nurse include in the client's plan of care? a.)Place the client in private room away from the nurses' station. b.)Search the client's room every 24 hours for unsafe objects. c.)Discourage the client from expressing negative emotions. d.)Develop a contract with the client that states they will not harm themselves.

d.)Develop a contract with the client that states they will not harm themselves.

The nurse is evaluating the plan of care for a client with benign prostatic hyperplasia (BPH). For which prescribed medication should the nurse notify the health care provider (HCP)? a.)Terazosin b.)Finasteride c.)Metoprolol d.)Diphenhydramine

d.)Diphenhydramine

The nurse is caring for a client who is recovering from a right total hip arthroplasty. The client reports a sudden onset of chest pain and difficulty breathing. What action should the nurse take first? a.)Notify the health care provider. b.)Obtain the client's vital signs. c.)Auscultate the client's lung fields. d.)Elevate the head of the bed.

d.)Elevate the head of the bed.

The nurse working on a surgical unit is caring for a client who had surgery earlier today. The client's blood pressure is 80/51 mmHg and the heart rate is 128 bpm. Which intervention should the nurse implement first? a.)Apply supplemental oxygen therapy. b.)Check the surgical dressing for bleeding. c.)Increase the rate of the IV fluid infusion. d.)Ensure the client has a patent airway.

d.)Ensure the client has a patent airway.

A client who is taking isoniazid for tuberculosis asks the nurse about the possible side effects of this medication. The nurse informs the client to report which side effect of this medication to the primary health care provider (HCP)? a.) Photosensitivity and photophobia b.)Double vision and visual halos c.)Confusion and light-headedness d.)Extremity tingling and numbness

d.)Extremity tingling and numbness

The nurse is admitting a 15-year-old client with a fracture of the arm that requires surgery. The adolescent is crying and unwilling to talk. Which action by the nurse would be most appropriate at this time? a.)Make arrangements for friends to visit. b.)Reassure the client that the surgery will go fine. c.)Try to distract the client with a computer game. d.)Give the client some privacy.

d.)Give the client some privacy.

The nurse is caring for a client who received 2 units of packed red blood cells after an episode of gastrointestinal bleeding. Which laboratory value should the nurse monitor closely? a.)Bleeding time b.)Platelets c.)White blood cells d.)Hematocrit

d.)Hematocrit

The nurse is caring for a client admitted with diabetic ketoacidosis (DKA). Which lab finding is the priority? a.)PaO2 79 mm Hg b.)pH 7.34 c.)Potassium 5.0 mEq/L d.)Hematocrit 60%

d.)Hematocrit 60%

The nurse is caring for a client newly diagnosed with chronic obstructive pulmonary disease (COPD). The nurse reviews the client's medical record and notes which risk factors? Select all that apply. a.)Hyperlipidemia b.)History of pulmonary embolus c.)Seasonal allergic rhinitis d.)History of childhood asthma e.)A family history of COPD f.)History of smoking tobacco products

d.)History of childhood asthma e.)A family history of COPD f.)History of smoking tobacco products

The client is a 16-year-old with full-thickness burns involving 20% total body surface area. After the initial 24 hours of treatment to replace fluids, which factor is used to determine if the client's fluid needs are being met? a.)Daily hematocrit results b.)Daily weight measurements c.)Parkland formula for fluids d.)Hourly urine output

d.)Hourly urine output

A client reports to the nurse the passage of hard dry stools at least twice a week. Which of these actions should the nurse suggest that the client take first to improve their bowel function? a.)Increase physical activity. b.)Avoid binding foods, such as cheese. c.)Use a chemical laxative as needed. d.)Increase daily fiber intake to at least 20 grams.

d.)Increase daily fiber intake to at least 20 grams.

A client is admitted with low T3 and T4 levels and an elevated thyroid stimulating hormone (TSH) level. On initial assessment, the nurse should anticipate which of these findings? a.)Tachycardia b.)Diarrhea c.)Heat intolerance d.)Lethargy

d.)Lethargy

The nurse is providing preoperative teaching for a client preparing for a thyroidectomy about the medication saturated solution of potassium iodide (SSKI, ThyroSheild) drops. Which information is important for the nurse to include? a.)Take the medication on an empty stomach. b.)Store the medication in the refrigerator. c.)The medication will enlarge the thyroid gland. d.)Mix the medication with juice or milk.

d.)Mix the medication with juice or milk.

The parents of a 4 year-old boy have just been informed that their son has a congenital neurologic disorder that is terminal. The nurse should anticipate the parents' reaction to fall into which crisis phase? a.)Resolution phase b.)Crisis phase c.)Impact phase d.)Pre-crisis phase

d.)Pre-crisis phase

The nurse is caring for a client with orders for complete bed rest. Which action by the nurse is most important in the prevention of the formation of deep vein thrombosis (DVT)? a.)Apply knee high support stockings b.)Elevate the foot of the bed c.)Encourage isometric leg muscle exercises d.)Prevent pressure at back of the knees

d.)Prevent pressure at back of the knees

The nurse is preparing interventions for a client with major depression who has been showing signs of impaired social interaction. Which intervention should the nurse implement initially? a.)Provide activities that improve concentration and focus. b.)Provide activities that include group interaction. c.)Provide activities that allow for decision-making. d.)Provide activities that require minimal concentration.

d.)Provide activities that require minimal concentration.

The home health nurse is preparing for a home visit of a new client. Which action is most important to ensure the safety of the nurse during the visit? a.)Observe for evidence of weapons in the home. b.)Carry a cell phone, pager and/or hand-held alarm. c.)Review documentation for previous entries about violence. d.)Remain alert and leave if cues suggest the home is not safe.

d.)Remain alert and leave if cues suggest the home is not safe.

The nurse is preparing a client for a pulmonary CT angiogram with contrast to rule out a pulmonary embolism. For which laboratory result should the nurse notify the health care provider immediately? a.)D-dimer level of 1.2 mcg/mL b.)Arterial blood gas PaO2 level of 80 mm Hg c.)Serum troponin level of 0.1 mg/mL d.)Serum creatinine level of 2.8 mg/dL

d.)Serum creatinine level of 2.8 mg/dL

A 68-year-old, postmenopausal female client has been prescribed tamoxifen for breast cancer with bone metastases. The nurse should teach the client about which potential adverse drug effect? a.)Symptoms of hypocalcemia b.)Seizures c.)Insomnia d.)Stroke-like symptoms

d.)Stroke-like symptoms

A nurse is assessing a client with a deep vein thrombosis. Which finding should the nurse anticipate? a.)Bilateral ankle edema b.)Rapid respirations c.)Chest pain d.)Swelling of one lower extremity

d.)Swelling of one lower extremity

The nurse is conducting a teaching session to new nurses about the principles of pain management. Which principle is most important when assessing a client's pain level? a.)Clients have the right to have their pain relieved. b.)Cultural sensitivity is fundamental to pain management. c.)Nurses should not prejudge a client's pain using their own values. d.)The client's self-report is their actual pain level.

d.)The client's self-report is their actual pain level.

The nurse is providing discharge teaching to the parents of a 15-month-old child diagnosed with Kawasaki disease. The child received intravenous immunoglobulin therapy during the hospitalization. Which information should the nurse include? a.)Active range of motion exercises should be done frequently. b.)High doses of aspirin will be continued for some time. c.)Complete recovery is expected within several days. d.)The measles, mumps and rubella vaccine should be delayed.

d.)The measles, mumps and rubella vaccine should be delayed.

The nurse is evaluating a stage III pressure ulcer. Which assessment finding would indicate that the prescribed treatment is working? a.)Soft yellow tissue seen in wound bed b.)The edge of the wound appears rolled or curled under c.)The periwound texture is moist and soft d.)The size of the wound is decreasing

d.)The size of the wound is decreasing

which nursing practice best reduces the chance of communication errors that could lead to negative client outcomes? a.)Maintain respectful working relationships with all staff b.)Document nursing care at the end of the shift c.)Speak using a professional tone on the telephone d.)Use standardized forms for client handoffs

d.)Use standardized forms for client handoffs


Conjuntos de estudio relacionados

Grade 8 Unit 14 Synonyms and Antonyms

View Set

M.A 106: CH 12 Electronic Health Records

View Set

Cardio Pharm Qs - CHF, Diuretics, Anti-anginal, Anti-arrhythmics, Lipid Lowering

View Set

Ch. 12 HW Quiz: Social Psychology

View Set